You are on page 1of 90

Contents

Preface
1. Pitfalls to be Taken Care of
2. Subject-Verb Agreement
3. Indianism
4. Correction of Common Errors (Nouns)
5. Correction of Common Errors (Pronouns)
6. Correction of Common Errors (Adjectives)
7. Correction of Common Errors (Adverbs)
8. Correction of Common Errors (Prepositions)
9. Correction of Common Errors (Conjunctions)
10. Correction of Common Errors (Articles)
11. Correction of Common Errors (Word-Order)
12. Typical Sentences for Correction
Preface

Correct English Usage is a unique and peerless book for those who are interested in having a
written and spoken command of the English language. The fascination of this international language
is unmatched. But it is replete with pitfalls which must be carefully avoided if one does not want
to become a butt of laughter either in written or spoken English. Let us quote from the book itself,
"A student of the English language ought to know the difference between 'English' and 'the
English'." On the face of it, both these words appear to be so similar that it is hard to tell the
difference between them. You will be amazed to know that there is a world of difference in their
meanings. 'English' means the language whose intricacies we are going to clarify in this book but
'the English' means the people of England who speak English. All such pitfalls regarding the use of
nouns, pronouns, adjectives, prepositions, verbs, adverbs, conjunctions and so on have been
explained in simple and easy to understand language.
This book is a treasure house of modern and current usage in English. As one flips through the
pages of the book, one gains in knowledge and confidence. The mystery of the English language
unfolds itself in every page of this wonderful treatise. Its is sure the illumine the eyes and minds of
readers.
This book will serve as a readymade guide (to correct grammatical concepts) to teachers and
students and enrich their knowledge of English. A careful study of this book will certainly remove
confusions regarding the correct usage of words.
Prashant Gupta
The English language is rich, varied beautiful and enticing at the same time. The fascination of
this international language knows no bounds. But it is replete with pitfalls which must be carefully
avoided if one does not want to become a laughing stock either in written or spoken English.
There are four basic rules in the English grammar. These rules are, in fact, the four pillars of the
English grammar. If these rules are taken care of, one is sure to gain the mastery of the English
language to a great extent. In fine, these rules are the tips that won't let you 'trip'.

Rule I
Can, Could, Will, Would, Shall, Should, May, Might, Ought to, Must, Do, Did, Does, Used to,
Dare, Needall these words except 'Do', 'Did' and 'Does' are auxiliary verbs or helping verbs or
subordinate verbs. They help the Principal Verbs or Main Verbs to form meaningful sentences.
Note that the first form of the main verb is ever used after each of these helping verbs, come
what may. Let us have a look at the usage of each of these auxiliary verbs:-

Can
It is used to indicate ability or physical strength:-
(1) He can climb this mountain.
(2) My grandpa can walk for a mile at a stretch.
(3) She can play football.
(4) We can solve this problem.
(5) She can move this heavy trunk by herself.

Note carefully that the first form of the Main Verb is used after 'can' in each of the sentences. Never
write or sayI can did this work. It is absolutely wrong. Always write or sayI can do this
work.
It is used to indicate acquired knowledge or skill:-
(1) She can play on the piano.
(2) We can dance very well.
(3) He can sing sweetly.
(4) She can cook food.
(5) My father can fly a plane.

It is used to indicate that someone is aware of something through one of his senses:-
(1) He can see with his eyes.
(2) She can hear with her ears.
(3) I can feel with my skin.
(4) You can taste with your tongue.
(5) One can smell with one's nose.

It is used to describe what is considered characteristic, what somebody or something is


considered capable of being or doing:-
(1) Tsunami can be lethal at times.
(2) The elephant can be furious at any time.
(3) It can be biting hot here in June and July.
(4) He can be eccentric at times.
(5) It can be windy as well as snowy on hills.

Could
It is used to indicate the past tense of 'can':-
(1) He ran so fast that he could catch the bus.
(2) He told me that he could read the novel.
(3) I could reach there on time.
(4) I could help him in distress.
(5) He worked so hard that he could stand first in the class.

It is used to indicate a polite request:-


(1) Could you tell me the way to the nearest inn?
(2) Could you lend me your bicycle for a day?
(3) Could you solve this sum?
(4) Could you postpone the meeting for a few days?
(5) Could you bring me a glass of water?
Please be noted that this use of 'could' is employed in interrogative sentences only.

It is used to indicate ability in the past:-


(1) When I was a child, I could drink a glass of milk in one draught.
(2) I could run for miles at a stretch when I was in my teens.
(3) When I was ten years old, I could solve crossword puzzles in the twinkling of an eye.
(4) She could play on the mouth-organ when she was six.
(5) My father could memorize ten new English words daily when he was in college.

Will
It is used with the second and third persons in future tense:-
(1) He will fly to America tomorrow.
(2) She will be ten next year.
(3) You will do this work in the evening.
(4) They will leave for Agra next Monday.
(5) Schools will reopen on Monday.

It is used with the second person in questions, making


polite requests, invitations, etc.:-
(1) Will you please do me a favour?
(2) Will you accompany me to the airport?
(3) Will you lend me a hundred rupees?
(4) Will you pass the salad?
(5) Will you please open the windows?
(6) Will you attend my marriage?

It is used with the first person to indicate determination,


threat, promise, intention, etc.:-
(1) I will teach you a bitter lesson.
(2) I will cross this river, come what may.
(3) I will buy a good book on the English grammar.
(4) We will stay here tonight.
(5) I will try to do better next time.

It is used with the third person in interrogative sentences:-


(1) Will it rain heavily tonight?
(2) Will he speak the truth?
(3) Will they fight with their neighbours?
(4) Will she go shopping in the evening?
(5) Will they swim in the afternoon?

Would
It is used as the past tense of 'will':-
(1) My father told me that he would take me to the zoo next Tuesday.
(2) He knew that he would need some money.
(3) I asked him if he would solve the sums.
(4) The teacher asked Mohan if he would pay the fine.
(5) I told her that I would help her in distress.

It is used to make polite requests and suggestions:-


(1) What would you take?
(2) I would take coffee with burger.
(3) Would you lend me a thousand rupees?
(4) Would you pay me by cheque?
(5) Would you accompany me to the station?

It is used to describe the consequences of an imagined event:-


(1) If you had your hair cut short, you would look handsome.
(2) If you behave gently, everybody would respect you.
(3) If I won a lottery, I would set-up a school in this village.
(4) If I owned a car, I would drive it by myself.
(5) If I were a bird, I would fly in the sky.

It is used to describe a condition in the past:-


(1) If I had seen him, I would have talked to him.
(2) If I had been there, I would have beaten him.
(3) If I had owned a car, I would have driven it by myself.
(4) If it had rained on time, the crops would not have been destroyed.
(5) If he had come to me, I would have helped him.

It is used to express preferences:-


(1) I would like to go there alone.
(2) I would prefer sugar to salt.
(3) I would prefer jogging to cycling.
(4) I would prefer coffee to tea.
(5) I would like to eat a sandwich.
It is used to comment on characteristic behaviour or habitual or repeated activities:-
(1) I would drink a lot of milk when I was young.
(2) I would go for a walk every morning.
(3) I would fast once a month when I was in college.
(4) I would quarrel a lot when I was a child.
(5) I would fly kites during my childhood.

Note that 'would' is equivalent to 'used to'.


It is used to make requests using wish:-
(1) I wish they would behave well.
(2) I wish he would not make a noise.
(3) I wish you would listen to your teacher.
(4) I wish you would be sincere.
(5) I wish she would sing sweetly.

Shall
It is used with the first person in future tense:-
(1) We shall go to a grand party in the evening.
(2) I shall give you a few tips on cooking.
(3) I shall go to America by plane tomorrow.
(4) I shall invite all my friends to tea.
(5) We shall go to the club tomorrow.

It is used with the second and third persons to indicate promise, threat, command, determination,
certainty, etc.:-
(1) Children, you shall have a holiday tomorrow.
(2) Our soldiers shall fight to the last.
(3) You shall speak the truth.
(4) You shall be fired if you don't behave properly.
(5) I shall be leaving as soon as I am called.

Should
It is used to indicate obligation or duty:-
(1) We should obey traffic rules.
(2) You should serve your ailing parents.
(3) He should observe discipline in the classroom.
(4) One should keep one's promise.
(5) We should always speak the truth.

It is used in indirect speech as the past form of 'shall':-


(1) The teacher advised the students that they should brush their teeth twice a day.
(2) The captain instructed the soldiers that they should fight to the finish.
(3) Mother advised her son that he should always speak the truth.
(4) My father told me that I should never tell a lie.
(5) My parents told me that we should pray to the Almighty daily without fail.

It is used to indicate a condition in the present tense:-


(1) Should you lose your way, ask somebody.
(2) Should you go out, post this letter.
(3) Should you miss the bus, go by train.
(4) Should you reach there safely, inform me.
(5) Should you play well, you are sure to win.
Note that 'should' is equivalent to 'if'.

It is used after conjunction 'lest':-


(1) Work hard lest you should fail.
(2) Work carefully lest you should stumble down.
(3) The pickpocket ran away lest he should be caught redhanded by the policeman.
(4) Speak softly lest you should be heard.
(5) Speak softly lest you should be heard by others.
Note that 'lest' is always followed by 'should' irrespective of the tense. In other words,
whatever is the tensepresent or past or future'lest' is always followed by 'should'.

May
It is used to indicate possibility:-
(1) It may rain tonight.
(2) It may hail today.
(3) It may snow today.
(4) I may come to see you in the evening.
(5) It may sleet tomorrow.

It is used to indicate permission:-


(1) May I borrow your bicycle for a day?
(2) May I go to the toilet, sir?
(3) May I go home now?
(4) May I wait for him?
(5) May I come in, madam?

It is used to indicate a purpose:-


(1) We eat so that we may live.
(2) We earn so that we may spend.
(3) He works hard so that he may get through.
(4) He runs fast so that he may catch the bus.
(5) He speaks aloud so that he may be heard by the audience.
Note that conjunction 'so that' is always followed by may + Ist form of verb in the present
tense. Never use 'can' or any other modal verb.

It is used to indicate a hope or wish:-


(1) May God grant you speedy recovery!
(2) May God bless you!
(3) May you scale great heights in life!
(4) May God grant you all the riches of the world!
(5) May you top the list of successful candidates!
Note that the sentences expressing wish or prayer also start with 'may'. But each of these
sentences ends with a mark of exclamation (!).

Might
It is used to indicate a purpose in the past tense:-
(1) He worked hard so that he might pass the examination.
(2) He ran fast so that he might catch the bus.
(3) He spoke aloud so that he might be heard by the audience.
(4) He earned so that he might spend.
(5) He flattered his boss so that he might be promoted.
Note that conjunction 'so that' is always followed by might + Ist form of verb in the past
tense. Never used 'could' or any other model verb.

It is used to indicate a lesser possibility:-


(1) It might sleet in the evening.
(2) It might snow tonight.
(3) It might rain today.
(4) It might hail in the afternoon.
(5) It might blow heavily tomorrow.

It is used as the past tense of 'may':-


(1) The teacher told the students that he might not come next day.
(2) He said that he might come next day.
(3) He told me that he might meet me on Tuesday.
(4) He threatened me that he might reveal the secret to the police.
(5) He informed me that he might give me some money next day.

It is used to ask permission politely:-


(1) Might I use your mobile phone?
(2) Might I use you pen?
(3) Might I sit down?
(4) Might I know who you are?
(5) Might I go home now?

Ought to
It is used to indicate moral obligation or duty:-
(1) A soldier ought to serve his motherland.
(2) We ought to serve our ailing parents.
(3) We ought to obey traffic rules.
(4) You ought to be punctual.
(5) She ought to do this work.
Note that both 'ought to' and 'should' can be used interchangeably. The former expresses
moral obligation whereas the latter expresses obligation.

Must
It is used to indicate necessity or compulsion:-
(1) You must run fast to catch the bus.
(2) You must reach school in time.
(3) You must see the doctor at once.
(4) You must hurry, or you will miss the train.
(5) You must burn midnight oil if you want to secure good marks.
(6) We must fight for liberty.
(7) You must seek forgiveness of your teacher.
(8) She must carry out my orders.
(9) You must clear the dues by the 10th of July.
(10) A servant must obey his master.

It is used to indicate inevitability:-


(1) All living things must eat to survive.
(2) All living things must grow.
(3) Everybody must die one day.
(4) All living things must breathe.
(5) What can't be cured, must be endured.

Dare
It is used to mean have sufficient courage or imprudence to do something:-
(1) He dare not stand up in public.
(2) I dare not ask him a question.
(3) How dare she say such nasty things about me?
(4) How dare you touch my book?
(5) She dare not cross this river.

It is used to indicate supposition. But it is used with 'say':-


(1) I dare say we shall be late for office.
(2) I dare say they will be in trouble.
(3) I dare say he will help me later.
(4) I dare say you are a foolish person.
(5) I dare say he is not a foreigner.
Note that 'dare say' is used in this way with the first person singular only.

Need
It is used to indicate an obligation:-
(1) Need I go now?
(2) You need not complete that work today.
(3) You need not go there to see her.
(4) You need not worry about that.
(5) You need not stay here any longer.

Used to
It is used to say that something happened continuously orfrequently during a period in the past:-
(1) I used to live in London.
(2) We used to go sailing on the lake in summer.
(3) I didn't use to like him much when we were at school.
(4) You used to see a lot of her.
(5) I used to drink a lot of milk when I was young.

Do, Does
They are used in the Simple Present Tense or Present Indefinite in affirmative sentences, negative
sentences, interrogative sentences and negative-interrogative sentences:-
(1) I do go to school daily.
(2) She does swim in the evening.
(3) They do not obey their elders.
(4) You do not tell a lie.
(5) Do you speak the truth?
(6) Don't you play cricket?
Note that in affirmative sentences 'Do and Does' are used to give emphasis to the main
verbs.

I go to school daily.
I do go to school daily.

There is a world of difference between the above two sentences. The first sentence is a
general statement whereas the second sentence asserts certainty without fail.

Did
It is used in the Simple Past Tense or Past Indefinite in affirmative sentences, negative sentences,
interrogative sentences and negative-interrogative sentences:-
(1) Did the teacher teach English yesterday?
(2) Did I not watch a movie yesterday?
(3) Did you not go to school yesterday?
(4) She did play on the flute yesterday evening.
(5) I did meet her in the street.
(6) He did not make a noise.
(7) They did not play well.
Note that in affirmative sentences 'did' is used to give emphasis to the main verb:

I watched a movie yesterday night.


I did watch a movie yesterday night.

The first sentence is a general statement while the second statement asserts certainty
without fail.
'Do, Does and Did' are not modal verbs; they are only auxiliary or helping verbs.

Rule II
Be, Is, Am, Are, Was, Were, Has, Have, Had, Been
All the above-mentioned words are again auxiliary or helping verbs. Some of these verbs are used
in both Active Voice and Passive Voice whereas some others are used in Passive Voice only. It is
to be noted that the third form of the main verb is used after these helping verbs.

Be
'Be' as an auxiliary verb is used as follows:
be + V3

Generally, it is used with 'let', i.e.


Let + object + be + V3

(1) Be seated.
(2) Let the food be served.
(3) Let a letter be written.
(4) Let the clothes be washed.
(5) Let the match be played.
(6) Let the over be bowled.
(7) Let the book be read.
(8) Let the lesson be revised.
(9) Let TV be watched.
(10) Let the car be driven.

Is, Am, Are


All these three helping verbs are used while making the passive voice of Simple Present Tense.
The rule is as follows:-
Sub + is/am/are + V3 + Obj.

(1) I am praised for my efforts.


(2) She is rebuked by her teacher.
(3) Books are written by me.
(4) They are insulted by their relatives.
(5) A page is torn by the child.

Was, Were
These two helping verbs are used while making the passive voice of Simple Past Tense. The rule
is as follows:-
"Sub + was/were + V3 + obj.

(1) The dog was run over by a speeding truck.


(2) The letter was written in haste.
(3) All the mangoes were eaten by the hungry beggar.
(4) The road was repaired by the labourers.
(5) The gifts were given away by the Principal.

Has, Have
Both these helping verbs are used in Present Perfect Tense both in Active Voice and in Passive
Voice. The rule for making Active Voice is as follows:-
"Sub + has/have + V3 + Obj.

(1) The child has broken the toy.


(2) He has done his homework.
(3) We have made all the preparations for journey.
(4) We have won the match.
(5) They have lost the game.

The rule for making Passive Voice is as follows:


Sub + has been/have been + V3 + Obj.
(1) The plants have been watered by the gardener.
(2) The cow has been milked by the milkman.
(3) The work has been completed before time.
(4) She has been helped by her friends.
(5) I have been adjudged 'Man of the Match'.

Had
This helping verb is used in Past Perfect Tense both in Active Voice and in Passive Voice. The
rule for making Active Voice is as follows:-
Sub + had + V,3 + Obj.
(1) The students had made a lot of noise.
(2) The teacher had taught the lesson.
(3) The lion had killed the deer.
(4) Mother had cooked food by then.
(5) I had reached my destination by evening.

The rule for making Passive Voice is as follows:-


Sub + had been + V3 + Obj.

(1) The speech had been delivered by the minister.


(2) The letter had been written and posted.
(3) The pencil had been mended.
(4) The shoes had been repaired.
(5) The house had been whitewashed.

Been
It is another helping verb which is used in Present Perfect,Past Perfect and Future Perfect in
Passive Voice only.
In Future Perfect the rule is as follows:-
Sub + will have been/shall have been + V3+ Obj.

The use of 'been' in Present Perfect and Past Perfect has already been stated under the helping
verbs 'has', 'have' and 'had'.
(1) The house will have been washed by 5 o'clock in the evening.
(2) The race will have been won by the tortoise.
(3) The village will have been looted by the dacoits.
(4) The project will have been completed by evening.
(5) The cow will have been milked by seven in the morning.

Rule III
Past Tense is followed by Past Tense. If the verb in the main or principal clause is in the past
tense, the verb in the subordinate clause is also in the past tense. Let us have a look at the
following sentences:-
(1) Ashoka wished that he might become a king.
(2) I knew that I could not get success.
(3) Subash told me that he had been ill.
(4) He hoped that his friend would help him.
(5) Kamal worked hard so that he might pass.
Note that in each sentence the verb in the past tense in the main clause is followed by the
verb in the past tense in the subordinate clause.

There are, however, a few exceptions to this rule. These exceptions are:
(a) If the subordinate clause states some habit or universal truth, the verb, in that case, in the
subordinate clause, is in the present tense, although the verb in the main clause is in the past
tense:-
(1) Galileo said that the earth moves round the sun.
(2) My father told me that the sun rises in the east.
(3) My mother told me that two and two make four.
(4) The teacher told the students that honesty is the best policy.
(5) My uncle told me that the earth has only one satellite.

(b) The verb of the subordinate Adverb clause of place, reason or comparison, may be in any
tense, as the tense requires it, even though the verb in the Principal Clause is in the past
tense; as,
(1) He did not pass the examination because he is weak.
(2) My mother liked me more than she likes you.
(3) He ran to the place where his friend lives.
(4) I could not help my friend because I have no money.
(5) The beggar could not cross the road as he is blind.

(c) The past tense in the Principal Clause may be followed by any tense, as required by the
sense, in the subordinate clause, if it is an Adjective clause; as,
(1) I called on the man who will take me in service.
(2) I came across the boy who has been my next-door neighbour.
(3) I knew the man who has killed my uncle.
(4) She met a girl who is well versed in calligraphy.
(5) They knew the beggar who owns a big house.

Rule IV
A verb agrees with its subject in number. In other words, we can say that a verb takes the same
number as its subject; as,
(1) The cow is a useful animal.
(2) Cows are useful animals.
(3) The boy reads in this class.
(4) The boys read in this class.
(5) The child is crying.
(6) The children are crying.

In the first sentence, 'cow' is in the singular number, so the verb 'is' also is in the singular number.
In the second sentence, the subject 'cows' is in the plural number, so the verb 'are' also is in the
plural number.
The same is the case with all other sentences.
A verb agrees with its subject in person as well. In other words, we can say that a verb changes its
form with a change in the person of the subject; as,
(1) I sing a song.
(2) He sings a song.
(3) They go to school.
(4) He goes to school.
(5) She swims daily.
(6) You swim daily.

In the first two sentences, the subjects 'I' and 'She' are singular in number. The subject in the first
sentence is, however, a pronoun of the first person while the subject in the second sentence is a
pronoun of the third person. The verbssingular or pluralhave been used accordingly.
Note: Rule IV is quite exhaustive; that is why we have made a separate chapter on it. Let us
study it further in the second chapter.
As stated in the previous chapter, the verb must agree with its subject in Number and Person.
Error of proximity: Sometimes a verb is made to agree in number and person with a noun near it
instead of with its own subject. It is a blunder and should be avoided. Let us have a look at the
following sentences:-
(1) The quality of these mangoes is (not, are) good.
(2) The introduction of reforms in education was (not, were) liked by all.
(3) A variety of beautiful roses charms (not, charm) the eye.
(4) The information in these newspapers is (not, are) not reliable.
(5) The announcement of liberal policies has (not, have) attracted general public.

Noun + of + nounin the sentence having this pattern, the noun coming before 'of' is
considered the subject and the verb comes accordingly.
Two or more singular subjects joined by 'and' take a plural verb; as,
(1) Kanta and her sister are sweet girls.
(2) The teacher and the student were present.
(3) Mohan and his brother are in the wrong.
(4) The elephant and the cow are grass-eating animals.
(5) Mother and her son are in the kitchen.

When two subjects joined by 'and' express one idea or refer to the same person or thing, the verb
is singular; as, (1) Bread and butter is his favourite food.
(2) Rice and curry is his favourite dish.
(3) Slow and steady wins the race.
(4) Pride and honour of the country is at stake.
(5) Buttermilk and Makki ki Roti is his favourite lunch.

When one of the two subjects joined by 'and' takes an article, the verb is singular; as,
(1) The great patriot and soldier is now dead.
(2) The horse and carriage is at the door.
(3) The Director and Chairman of the Board of Education is a learned man.
(4) The secretary and treasurer of the club has arrived on time.
(5) The captain and wicket keeper of the Indian team is a nice gentleman.

When both the subjects joined by 'and' take articles, the verb is plural; as,
(1) The King and the Queen were struck with grief.
(2) The President and the Vice-President are sitting in the Parliament Hall.
(3) The black and the white cow are grazing in the field.
(4) The first and the last chapter of this book are boring.
(5) The principal and the teacher were discussing the matter.

The distributive determinerseach and everysingle out individual persons. These determiners
take singular verbs; as,
(1) Each boy and each girl in the school is dear to the principal.
(2) Everyone present was amazed at the feat of the magician.
(3) Every person in the country has a right to vote.
(4) Each participant was given a consolation prize.
(5) Every soldier owes a debt of gratitude to his motherland.

When two nouns are joined by 'as well as' 'not' or 'with', the verb agrees in number and person
with the first subject; as,
(1) The King as well as his Chief Minister has arrived.
(2) Krishna, and not you, deserves (not, deserve) the prize.
(3) The Prime Minister with his Cabinet Ministers is expected
to see the Industrial Exhibition.
(4) The teacher as well as the students is going to visit the zoo.
(5) The children with their parents have entered the Rail Museum.

When two subjects are joined by 'not onlybut also', the verb agrees in number and person with
the second subject; as,
(1) Not only the students but also the Principal was (not, were) given sweets.
(2) Not only the Principal but also the students were (not, was) given sweets.
(3) Not only the coach but also the players were (not, was) in the playground.
(4) Not only the players but also the coach was (not, were) found guilty.
(5) Not only the Ministers but also the Prime Minister was (not, were) charged with bribery.

A singular verb follows 'each', 'every', 'either', 'neither' and 'many a'; as,
(1) Each of them was awarded a prize.
(2) Every man, woman and child in the house on fire has (not, have) been saved.
(3) Many a flower is (not, are) born to blush unseen.
(4) Either of these two persons has stolen the purse.
(5) Neither of these two girls is intelligent.
(6) Everyone of them is gentle.
Note carefully that 'many a' is equivalent to 'many' in meaning. But it takes a singular verb.
Remember, 'many a flower' means 'many flowers'.

When two subjects are joined by 'eitheror', 'neither nor' and 'or', the verb is, in each case, in
the singular number; as,
(1) Neither Ramesh nor Suresh is my brother.
(2) Neither the teacher nor the student is to be blamed.
(3) Harish or Suresh is in the wrong.
(4) This cloth or that one is required at once.
(5) Either Mohan or his friend has stolen my wallet.

When two subjects are joined by 'eitheror' or 'neither nor' and one of them is in the plural
number, the plural subject must be placed nearest the verb, and the verb must also be plural; as,
(1) Neither Mr. Goel nor his friends have been invited to the dinner.
(2) Either Prabha or her parents are in the wrong.
(3) Neither the teacher nor the students are in the classroom.
(4) Either the President or the MPs have been found guilty of misbehaviour.
(5) Either Mohan or his parents are to receive the honour.
When two subjects of different persons are joined by 'either-or' or 'neither-nor', the verb agrees
in person with the subject nearest to it; as,
(1) Either he or you have (not, has) told a lie.
(2) Either they or she is in the wrong.
(3) Neither you or I have done this blunder.
(4) Neither he nor I like (not, likes) to be dishonest.
(5) Either she or you are to be punished.

A collective noun, in the singular, may take the verb in the singular or in the plural according as
the speaker thinks of the thing as a single whole or of the individuals of which it is composed; as,
(1) The crowd (i.e. as a single body) has assembled very soon.
(2) The crowd (i.e. the individuals of the crowd) were lathicharged by the police.
(3) The Government (i.e. as a single body) has introduced a new legislation.
(4) The Government (i.e. the members of the Government) have tried to solve the intricate
problem.
(5) The jury (i.e. the individuals of the jury) were divided in their opinions.
Note that there are so many collective nouns, such as audience, board, team, class and so
on.

When the plural subjects denote some particular amount of quantity considered as a whole, the
verb is in the singular number; as,
(1) Eight kilometres is a long distance.
(2) A hundred paise is equal to one rupee.
(3) A hundred cents is equal to one dollar.
(4) A thousand years is called a millennium.
(5) Ten years is called a decade.

When the plural noun is a proper noun for some single subject or some collective unit, the verb is
in the singular number; as,
(1) Glimpses of World History is a famous book.
(2) The United States of America has done marvellous progress.
(3) Aesop's Fables is an interesting book.
(4) Arabian Nights is a wonderful book.
(5) Satanic Rites is (not, are) a unique write-up.

Nouns which are singular in form but plural in meaning take plural verbs; as,
(1) Six dozen bananas cost only Rs. 240.
(2) A few dozen people were there in the week.
(3) Two score make forty.
(4) Scores of cars were parked outside.
(5) Dozens of eggs were thrown at the speaker.

Some nouns, though plural in form, are treated as singular; as,


(1) The wages of sin is death.
(2) This news is too good to be true.
(3) Measles is a deadly disease.
(4) Mathematics is an interesting subject.
(5) Civics is about the government and its rule.
When a relative pronoun is the subject of a verb, the verb agrees with its antecedent in number
and person; as,
(1) You, girl, are (not, is) very delicate.
(2) You, who are a good writer, must compete for the prize.
(3) She, who is dressed in white, is our teacher.
(4) I, Ram Lal, am (not, is) a politician.
(5) I, who am now penniless, can't help you.

A compound subject formed by joining singular nouns by 'and' takes a singular verb; as,
(1) 'Hammer and sickle' is there.
(2) 'Screaming and shouting' was heard.
(3) 'The roof and crown' of the world is man.
(4) 'Coming and going' is not allowed when the show is on.
(5) 'Scolding and beating' is totally prohibited in this school.

If a countable noun comes after the wordsall and someit is in plural, so it takes a plural
verb; as,
(1) All the buildings are high.
(2) Some apples are rotten.
Note that building is a countable noun. So after the word 'all' it comes in plural, i.e. buildings.

If an uncountable noun comes after the wordsall and someit takes a singular verb; as,

(1) All the wealth was taken away.


(2) Some gold is stolen.

If the word 'all' denotes everything, it takes a singular verb; as,


I will tell you all that was (not, were) negotiated.
'Many of', 'a few of', 'two of', 'several of', 'the majority of'take plural subjects and plural verbs;
as,
(1) Many of the boys were absent yesterday.
(2) The majority of the boys were guilty.
(3) A few of my classmates are selfish.
(4) Two of the teachers in my school are blind.

If 'the majority' forms the subject, it can take a singular verb; as,
The majority was (not, were) against the circular.

The wordsadvice, furniture, apparatus, equipment, information, weather, work and so onare
singular and take singular verbs. We can't add 's' or 'es' to them to get their plural forms. To use
such nouns, in plural, we put before them 'pieces of', 'kinds of', 'branches of' and so on; as,
(1) These pieces of information/advice are useful.
(2) Different branches of knowledge are there in this treatise.

When the words 'politics', 'economics', 'mathematics', and so on are treated as plural, they mean
political calculations, economical facts and mathematical calculations respectively. So, they take
plural verbs; as,
(1) His politics are accurate.
(2) My mathematics are very weak.
(3) The economics of the project are very encouraging.

'More than' followed by a singular subject takes a singular verb; as,


(1) More than one prisoner has complained.
(2) More than one student has been appreciated.

When we place the definite article 'the' before an adjective, we get a plural noun which takes a
plural verb; as,
(1) The rich are respected.
(2) The poor are humiliated.
(3) The rich hate (not, hates) the poor.
Note well that rich or poor is an adjective whereas 'the rich' or 'the poor' is a plural noun.

If the subject in a sentence contains two clauses, it takes a plural verb; as,
(1) What I say and think are similar.
(2) Your behaviour and mine are peculiar.
Note that the first sentence has two clauses, i.e. what I saywhat I think. The same is the
case with the second sentence.

Let us study a few sentences:-


(1) A half of it is OK.
(2) A half of them are (not, is) OK.
(3) Two-thirds of the area is irrigated.
(4) Two-thirds of the boys are absent.
(5) Ten per cent of the spectators were present.
(6) Ten per cent of the amount is wasted.
(7) The rest of the job is done.
(8) The rest of the assignments are not done.

In such sentences, the verb (singular or plural) is used as per the noun following the word 'of'.

If the word 'percentage' comes before the word 'of', we always employ a singular verb; as,
(1) The percentage of naughty boys is very high.
(2) Some percentage of my income is (not, are) deducted at source.

In a sentence that has the patternsingular noun + preposition + the same noun repeatedwe use
a singular verb; as,
(1) Car after car is passing this way.
(2) Book after book describes her beauty.
(3) One hour after another has passed.

The word 'other' takes a plural subject and a plural verb. The word 'another' takes a singular
subject and a singular verb; as,
(1) No other buildings are like ours.
(2) Another building is under repairs.
(3) Other buildings in this area have been constructed as per the structural safety guidelines.

'Tools' and 'articles of dress' made of two equal parts are considered as plural. So, they take
plural verbs; as,
(1) His trousers are new.
(2) Her shoes are dirty.
(3) My glasses are dirty.
(4) The tongs are terribly hot.
(5) Scissors are to cut with.
(6) My socks are blue in colour.
(7) Her compasses are accurate.
(8) My dividers are broken.
(9) His shorts are loose.
(10) Her knickers are too short.
Note well that when these plural words are preceded by 'pair of', each of them takes a
singular verb; as,

(1) This pair of shoes is (not, are) costly.


(2) That pair of trousers over there is (not, are) made of denim.
(3) This pair of compasses is (not, are) made of steel.
(4) A pair of guns is (not, are) required at once.

If the word 'nothing' is the subject of a sentence, we employ a singular verb even if 'nothing but'
is followed by a singular or a plural noun; as,
(1) Nothing but participants was seen.
(2) Nothing but bushes is seen.
(3) Nothing but water was seen.
(4) Nothing but a pall of gloom has descended upon the town.
(5) Nothing but cries was heard.

Some words, though singular in form, are treated plural and take plural verbs; as,
(1) Cattle are grazing in the field.
(2) Deer are fast runners.
(3) Fish dwell in water.
(4) Sheep live on grass and fodder.
Note that 'deer', 'fish' and 'sheep' have the same singular and plural forms. When 'fish' is
preceded by a numeral, it may be written as 'two fishes, three fishes' and so on.

When the word 'none' is followed by an uncountable noun, the verb is singular. When it is
followed by a plural subject, the verb may be singular or plural; as,
(1) None of the information is reliable.
(2) None of the trains go/goes (both are correct) to Kalka.

When the word 'means' means 'a way to an end', thisword is used with either a singular or a
plural verb as per the words before it. When the word 'means' means 'money or material
possessions', it takes a plural verb; as,
(1) This means is (not, are) tried so far.
(2) These means are (not, is) not tried.
(3) My means are (never, is) are scanty.

The word 'riches' means material possessions. So, it always takes a plural verb; as,
(1) Riches have wings.
(2) Riches change hands quickly.
(3) Riches make or mar a person.

Similarly, the word 'alms' which means 'money, clothes and food' also takes a plural verb; as,
Alms were given to the poor.

'One of' is always followed by a plural subject. But it takes a singular verb; as,
(1) One of my uncles lives in the USA.
(2) One of my friends is very rich.
(3) One of the reports is not reliable.

In imaginative sentences, 'I' is followed by were, not by was; as,


(1) If I were (not, was) a bird, I would fly high in the sky.
(2) If I were in your place, I would have kicked him out of the house.

When 's' is added to the word 'people', it becomes peoples which means 'nations'. So, it takes a
plural verb; as, The peoples of the world are (not is) coming closer thanks to the Internet.
The word 'poultry' may take either a singular verb or a plural verb depending on its meaning; as,
(1) The poultry is (not, are) expensive nowadays.
(2) The poultry are (not, is) reared on this farm.
Note well that in the first sentence, 'poultry' means 'flesh of ducks, geese,' etc. In the second
sentence, 'poultry' means 'ducks', geese, etc.

The word 'gentry' means 'people of high class'. It always takes a plural verb; as,
(1) The gentry of the town were given a warm welcome.
(2) The gentry were divided in their opinions over the matter of corruption.
Similarly, the word 'clergy' means 'priests or ministers of a religion'. It always takes a plural verb;
as, All the local clergy were (not, was) asked to attend the ceremony.
(a) The murderer was caught and hung. (Incorrect)
The murderer was caught and hanged. (Correct)
(b) He hanged the picture. (Incorrect)
He hung the picture. (Correct)
Note: 'Hang' when it has its formshanged, hanged means 'to give capital punishment'
whereas 'hang', when it has its formshung, hungmeans 'to attach something at the top so
that the lower part is free or loose'.

(a) America was invented by Columbus. (Incorrect)


America was discovered by Columbus. (Correct)
(b) Edison discovered the gramophone. (Incorrect)
Edison invented the gramophone. (Correct)
(c) The man gave a speech. (Incorrect)
The man made a speech. (Correct)
(d) He made a lecture. (Incorrect)
He delivered/gave a lecture. (Correct)
Note: Use 'discover' for a thing, which already exists but is unnoticed. 'Make a speech' and
'deliver or give a lecture' are set expressions.
By 'Indianism' we mean all those expressions which are essentially un-English but which are
generally regarded as 'correct' by the so-called Indian students of English. In most cases, they are
the literal translation of some native idiom or phrase. They are, in fact, the distortions of the
correct English expressions and idioms. These queer expressions have given rise to what is known
as Indian English or 'Babu English'. The 'King's English' or the 'Queen's English' is considered to
be absolutely flawless. It is the English language which is written and spoken impeccably by
educated people in the United Kingdom.
Let us discuss a few examples of Indianism and their correct usage in our everyday life.
(1) 'Work' and not 'Works'
We are responsible to God for our works. (Incorrect)
We are responsible to God for our deeds or work. (Correct)
I have to attend to many works at home. (Incorrect)
I have to attend to many things at home. (Correct)
He performed many brave works. (Incorrect)
He performed many brave deeds. (Correct)
Note that 'works' in the plural means books or a factory; as,

Works of Shakespeare, an engineering works or a brickworks


(2) 'Tell a lie' and not 'speak a lie'
Don't speak a lie. (Incorrect)
Don't tell a lie. (Correct)
(3) 'In good health' and not 'good in health'
He is good in health. (Incorrect)
He is in good health. (Correct)
(4) 'Ascended' and not 'ascended up'
He ascended up the mountain. (Incorrect)
He ascended the mountain. (Correct)
(5) 'To make both ends meet' and not 'to make both his ends meet'
He is so poor that he can't make both his ends meet. (Incorrect)
He is so poor that he can't make both ends meet. (Correct)
(6) 'Aim at' and not 'aim to'
We aim to publish good books. (Incorrect)
We aim at publishing good books. (Correct)
(7) '5.30 train' and not '5.30 o'clock train'
I am leaving for Agra by 5.30 o'clock train. (Incorrect)
I am leaving for Agra by 5.30 train. (Correct)
(8) 'Tony Blair, Esq.' and not 'Mr. Tony Blair, Esq.'
Mr. Tony Blair, Esq., is our Chief Guest. (Incorrect)
Tony Blair, Esq. is our Chief Guest. (Correct)
Note well'Esquire' or Esq. in short is a polite title written after a man's name, especially
on an official letter addressed to him. If 'Esq.' is used, 'Mr.' is not used.
(9) 'Cowardly' and not 'coward man'
He is a coward man. (Incorrect)
He is a cowardly man. (Correct)
(10) 'Yours sincerely' and not 'your sincerely' or your's sincerely
Your's or Your sincerely Mahesh Gupta (Incorrect)
Yours sincerely (Correct) Mahesh Gupta
(11) 'How do you do' and not 'quite well, thank you'
How do you do? (Incorrect) Quite well, thank you.
How do you do? (Correct) How do you do?
Note: The correct reply is to repeat 'How do you do' only.

12. 'The whole' and not 'whole'


Whole town came to receive him. (Incorrect)
The whole town came to receive him. (Correct)
13. 'All these' and not 'these all'
These all men are guilty. (Incorrect)
All these men are guilty. (Correct)
14. 'Lover of Nature' and not 'Lover of the Nature'
William Wordsworth was a lover of the Nature. (Incorrect)
William Wordsworth was a lover of Nature. (Correct)
15. 'Mankind' and not 'the mankind'
The mankind is thankful to the scientist who invented the television. (Incorrect)
Mankind is thankful to the scientist who invented the television. (Correct)
16. 'Man' and not 'the man'
The man is mortal. (Incorrect)
Man is mortal. (Correct)
17. 'A noise' and not 'noise'
Don't make noise. (Incorrect)
Don't make a noise. (Correct)
18. 'Scratch or erase' and not 'cut'
He cut that word. (Incorrect)
He scratched or erased that word. (Correct)
19. 'Dispose of' and not 'dispose off'
He has disposed off his old TV-set. (Incorrect)
He has disposed of his old TV-set. (Correct)
20. 'The shoulder' and not 'my shoulder'
He caught me by my shoulder. (Incorrect)
He caught me by the shoulder. (Correct)
21. 'Take the examination' and not 'give the examination'
I have given an MBA examination. (Incorrect)
I have taken an MBA examination. (Correct)
22. 'Avail oneself of' and not 'avail of'
I shall certainly avail of this opportunity. (Incorrect)
I shall avail myself of this opportunity. (Correct)
23. 'Preferable to' and not 'preferable than'
This plan is preferable than that. (Incorrect)
This plan is preferable to that. (Correct)
24. 'Scenery' and not 'sceneries'
Kashmir is famous for its sceneries. (Incorrect)
Kashmir is famous for its scenery. (Correct)
25. 'Advice' and not 'advices'
The teacher gave me many advices on the parting day. (Incorrect)
The teacher gave me much useful advice on the parting day.(Correct)
or
The teacher gave me many pieces of advice on the parting day.(Correct)
26. 'Felt' and not 'became'
On hearing the verdict he became very angry. (Incorrect)
On hearing the verdict he felt very angry. (Correct)
27. 'Cry' and not 'weep'
She was weeping because she had lost the way. (Incorrect)
She was crying because she had lost the way. (Correct)
28. 'Returned home' and not 'returned to home'
In the evening all of us returned to home. (Incorrect)
In the evening, all of us returned home. (Correct)
29. 'On the way' and not 'in the way'
In the way, I met an old friend. (Incorrect)
On the way, I met an old friend. (Correct)
30. 'With a view to doing' not 'with a view to do'
With a view to do this work I went there. (Incorrect)
With a view to doing this work I went there. (Correct)
Note: 'with a view to' is always followed by a gerund, not by an infinitive.

31. 'Members of one's family' and not 'family members'


Next day all my family members came to see me in the hostel. (Incorrect)
Next day all the members of my family came to see me in the hostel. (Correct)
32. 'Not to speak of' and not 'what to speak of'.
What to speak of French, he does not know even English. (Incorrect)
Not to speak of (or to say nothing of) French, he does not know even English. (Correct)
33. 'Places worth seeing' and not 'worth-seeing places'
There are many worth-seeing places in London. (Incorrect)
There are many places worth seeing in London. (Correct)
34. 'A five-rupee note' and not 'a five-rupees note'
He borrowed a five-rupees note from me. (Incorrect)
He borrowed a five-rupee note from me. (Correct)
35. 'Do what is necessary' and not 'do the needful'
I hope you will do the needful. (Incorrect)
I hope you will do what is necessary. (Correct)
36. 'Inferior to' and not 'inferior than'
This ink is inferior than that. (Incorrect)
This ink is inferior to that. (Correct)
37. 'Resemble' and not 'resemble with' or 'resemble to'
He resembles to/with his father. (Incorrect)
He resembles his father. (Correct)
38. 'Discuss' and not 'discuss about' or 'discuss on'
We discussed about/on the widow remarriage. (Incorrect)
We discussed the widow remarriage. (Correct)
39. 'Pulling well together' and not 'pulling on well'
Ramesh and Rajesh are pulling on well. (Incorrect)
Ramesh and Rajesh are pulling well together. (Correct)
Note: 'Pulling well together' means they are working in harmony; 'pulling on well' in the
sense of 'getting on nicely' is not a correct expression.

40. 'Ill' and not 'sick'


He fell sick yesterday. (Incorrect)
He fell ill yesterday. (Correct)
Note: 'Sick' is used when the illness has vomiting or one of its symptoms. Thus 'he is sick'
meanshe is unwell and he vomits. 'Sick' is metaphysically used in the sense of tired; as, I
am sick of this work.

41. 'Yet' and not 'but'


Though he is poor but he is honest. (Incorrect)
Though he is poor yet he is honest. (Correct)
42. 'How to swim' and not 'swimming'
He knows swimming. (Incorrect)
He knows how to swim. (Correct)
43. 'Went' and not 'had been'
I had been to Delhi last Monday. (Incorrect)
I went to Delhi last Monday. (Correct)
44. 'Has been working' and not 'is working'
Mahesh is working in the department for the last thirty years. (Incorrect)
Mahesh has been working in this department for the last thirty years. (Correct)
Note: When an action begins in the past and continues into the present, present perfect
continuous tense should be used.

45. 'Into' and not 'in'


She jumped in the river to save the drowning child. (Incorrect)
She jumped into the river to save the drowning child. (Correct)
46. 'Reach' and not 'reach at'
He reached at the station in time. (Incorrect)
He reached the station in time. (Correct)
47. 'On' and not 'from'
The winter term begins from first October. (Incorrect)
The winter term begins on first October. (Correct)
48. 'Marry' and not 'marry with'
Rama married with Sita. (Incorrect)
Rama married Sita. (Correct)
49. 'To one's address' and not 'on one's address'
Send this parcel on my address. (Incorrect)
Send this parcle to my address. (Correct)
50. 'Have been' and not 'am'
I am ill since Sunday. (Incorrect)
I have been ill since Sunday. (Correct)
51. 'Owing to' and 'due to'
Due to bad weather he could not come. (Incorrect)
Owing to bad weather he could not come. (Correct)
52. 'Pray to' and not 'pray'
He prays God every morning. (Incorrect)
He prays to God every morning. (Correct)
53. 'Different from' and not 'different to'
This is different to that. (Incorrect)
This is different from that. (Correct)
54. 'Who' and not 'and who'
He is a noble man and who can be trusted. (Incorrect)
He is a noble man who can be trusted. (Correct)
55. 'Have' and not 'have got'
I have got a spare pencil. (Incorrect)
I have a spare pencil. (Correct)
56. 'Notebook' or 'exercise book' and not 'copy'
Please write this letter in your copy. (Incorrect)
Please write this letter in your notebook. (Correct)
57. 'To' and not 'for'
This milk is good for drinking. (Incorrect)
This milk is good to drink. (Correct)
58. 'Fruit' and not 'fruits'
We bought some fruits at Shimla. (Incorrect)
We bought some fruit at Shimla. (Correct)
59. 'Vegetables' and not 'vegetable'
He prefers vegetable to beef. (Incorrect)
He prefers vegetables to beef. (Correct)
60. Double negative with 'until'
Until you don't work hard, you will not pass. (Incorrect)
Until you work hard, you will not pass. (Correct)
Note: In the sentence containing 'until', never use 'not' in that sentence.

61. 'Pick a quarrel' and not 'pick up a quarrel'


He picks up a quarrel with everybody. (Incorrect)
He picks a quarrel with everybody. (Correct)
62. 'Whether' and not 'as to whether'
I don't know as to whether he knows German. (Incorrect)
I don't know whether he knows German. (Correct)
63. 'Order' and not 'order for'
He ordered for a drink. (Incorrect)
He ordered a drink. (Correct)
64. 'Very' and not 'too'
I am too glad to see you. (Incorrect)
I am very glad to see you. (Correct)
65. 'Very' and not 'very much'
I am very much sorry. (Incorrect)
I am very sorry. (Correct)
I am very much pleased to befriend you. (Incorrect)
I am very pleased to befriend you. (Correct)
He is very much popular. (Incorrect)
He is very popular. (Correct)
66. 'Furniture' and not 'furnitures'
He sold all his furnitures. (Incorrect)
He sold all his furniture. (Correct)
67. 'The poor' and not 'poors'
You should be kind to poors. (Incorrect)
You should be kind to the poor. (Correct)
Note: 'The poor' means all the poor people.
68. 'Take leave of' and not 'take leave from'.
The headmaster took leave from his old students. (Incorrect)
The headmaster took leave of his old students. (Correct)
69. 'Part from a person' and not 'part with a person'
When I parted with my friends, I felt sad. (Incorrect)
When I parted from my friends, I felt sad. (Correct)
70. 'Part with a thing' and not 'part from a thing'
The miser was unwilling to part from gold. (Incorrect)
The miser was unwilling to part with gold. (Correct)
71. 'Affect' and not 'effect'
Overwork has effected his health. (Incorrect)
Overwork has affected his health. (Correct)
Note: 'Affect' is a verb while 'effect' is a noun.

72. 'No' and not 'yes'


Do you smoke? Yes, I don't. (Incorrect)
Do you smoke? No, I don't.(Correct)
or
Do you smoke? Yes, I do.(Correct)
Note: When the answer is in the negative, preface it with 'No'. When it is in the affirmative,
preface in with 'Yes'.

73. 'Speaking terms' and not 'talking terms'


I am not on talking terms with Harish. (Incorrect)
I am not on speaking terms with Harish. (Correct)
74. 'Women' and not 'females'
Many females were present at the meeting. (Incorrect)
Many women were present at the meeting. (Correct)
75. 'Enjoying good health' and not 'keeping good health'.
I hope you are keeping good health. (Incorrect)
I hope you are enjoying good health.(Correct)
or
I hope you are well.(Correct)
76. 'Enjoy himself' and not 'enjoy'
He went to the river and enjoyed. (Incorrect)
He went to the river and enjoyed himself. (Correct)
Note: The verb 'enjoy' being a transitive verb must have an object. Thus we should say:-
We enjoyed a holiday.
He enjoyed the scene before him.
He enjoyed himself.
77. 'Fear' and not 'hope'
I hope I will catch cold. (Incorrect)
I fear I will catch cold. (Correct)
78. 'Men in one's family' and not 'the male members of one's family'
The male members of my family were opposed to this idea. (Incorrect)
The men in my family were opposed to this idea. (Correct)
79. 'May I have a look at this' and not 'Can I have a look at this'
Can I have a look at this book? (Incorrect)
May I have a look at this book? (Correct)
Note: 'May' implies permission.

80. 'Good evening' and not 'good night'


Good night! How do you do? (Incorrect)
Good evening! How do you do? (Correct)
Note: 'Good night' is a parting salutation; when two people meet for the first time in the
evening, they should bid 'good evening' to each other.

81. 'Comes of' and not 'comes off'


He comes off a noble family. (Incorrect)
He comes of a noble family. (Correct)
Note: 'Come off' means 'to take place'; as,
His marriage comes off next Monday.

82. Superfluous 'as to'


I am undecided as to whether to accept this job or not. (Incorrect)
I am undecided whether to accept this job or not. (Correct)
83. 'What I am to do' or 'I was to do' and not 'what to do'
I did not know what to do. (Incorrect)
I did not know what I was to do. (Correct)
84. 'Look forward to expecting' and not 'look forward to expect'
I look forward to expect a positive reply from your side. (Incorrect)
I look forward to expecting a positive reply from your side. (Correct)
85. 'Help someone (to) cross' and not 'help someone in crossing'
I helped the blind man in crossing the road. (Incorrect)
I helped the blind man (to) cross the road. (Correct)
The students of the English Grammar commit blunders while speaking or writing English. They
do so because they lack the rudiments of the English Grammar. They are not fully familiar with the
subtle points of the English Grammar. Thus they fail to leave an everlasting impression on the
minds of readers or listeners through their conversation or writing. In the chapters to follow, we
have mentioned typical errors committed by students while using the Part of SpeechNouns,
Pronouns, Adjectives, Determines, Adverbs, Verbs, Prepositions, Conjunctions and Interjections.
They can avoid the errors if they grasp the underlying grammatical rules thoroughly.
1. The cattles are grazing in the field. (Incorrect)
The cattle are grazing in the field. (Correct)
Note: Some nouns have the same singular and plural forms; as, cattle, cannon, deer, sheep,
swine, etc.

2. He gave us two dozens mangoes. (Incorrect)


He gave us two dozen mangoes. (Correct)
Note: Some nouns when used after numerals have the same form in the plural as in the
singular; as, pair, dozen, gross, score, thousand, etc.
But we say: Hundreds of people gathered outside the Prime Minister's Office.

3. Your spectacle is fine. (Incorrect)


Your spectacles are fine. (Correct)
Note: Some nouns are used in the plural and always take plural verbs; as, scissors,
spectacles, trousers, etc.

4. Mathematics are a difficult subject. (Incorrect)


Mathematics is a difficult subject. (Correct)
Note: Some nouns are plural in form but they are used as singular; as, mathematics, news,
summons, the United States of America, etc.

5. The gentry of the town was invited. (Incorrect)


The gentry of the town were invited. (Correct)
Note: Some collective nouns are in singular form but they are used as plurals; as, people,
gentry, clergy, etc.

6. The sceneries of Kashmir are charming. (Incorrect)


The scenery of Kashmir is charming. (Correct)
Note: Some nouns are generally used in the singular form; as, advice, poetry, abuse,
furniture, luggage, scenery, etc.

7. (a) He is very fond of vegetable. (Incorrect)


He is very fond of vegetables. (Correct)
(b) He is very fond of fruits. (Incorrect)
He is very fond of fruit. (Correct)
Note: 'Vegetables' is always used in the plural form but 'fruit' in singular unless different
kinds of fruits are meant. The word 'fruits' means reward as well.

For exampleHe enjoyed the fruits of all his hard work.


8. (a) My hairs are black. (Incorrect)
My hair is black. (Correct)
(b) He has black and grey hair on his head. (Incorrect)
He has black and grey hairs on his head. (Correct)
(c) This tank teems with fishes. (Incorrect)
This tank teems with fish. (Correct)
(d) There are different kinds of fish in the sea. (Incorrect)
There are different kinds of fishes in the sea. (Correct)
Note: 'Hair' and 'fish' are used both in singular and plural forms.

9. (a) The jury is divided in their opinions. (Incorrect)


The jury are divided in their opinions. (Correct)
(b) The jury was unanimous in their decision. (Incorrect)
The jury was unanimous in its decision. (Correct)
Note: Some nouns are used both in the singular and in the plural; as, crowd, jury,
government, etc. We use a singular verb with a collective noun when it is thought to be one
whole, otherwise a plural verb.

10. He gave me ten-rupees note. (Incorrect)


He gave me a ten-rupee note. (Correct)
Note: The correct expressions are a ten-rupee note, a threefoot ruler, a seven-year old child,
a five-year plan and so on.

11. Death lays her icy hands on the rich and the poor alike. (Incorrect)
Death lays his icy hands on the rich and the poor alike. (Correct)
Note: Lifeless objects known for strength and violence are said to be of the Masculine
Gender, when they are personified; as, sun, winter, death, etc. Lifeless objects known for
beauty and gracefulness are said to be of the Feminine Gender, when they are personified;
as, moon, Nature, peace, etc.

12. I shall go by the 7.30 o'clock train. (Incorrect)


I shall go by the 7.30 train. (Correct)
13. We had a good play of hockey. (Incorrect)
We had a good game of hockey. (Correct)
14. We should help the poors.
We should help the poor.
Note: When an adjective is preceded by the definite article

'the', it becomes a plural noun.


15. The road is closed for repair. (Incorrect)
The road is closed for repairs. (Correct)
16. The magistrate passed an order for his arrest. (Incorrect)
The magistrate passed orders for his arrest. (Correct)
17. We have finished three-fourth of this book. (Incorrect)
We have finished three-fourths of this book. (Correct)
18. I have many works to do. (Incorrect)
I have much work to do. (Correct)
Note: 'Work' is an uncountable noun. When it is written 'works', it means factory.

19. There is no place in this compartment. (Incorrect)


There is no room in this compartment. (Correct)
20. Give me some blotting. (Incorrect)
Give me some blotting paper. (Correct)
21. She is one of the best girl in the class. (Incorrect)
She is one of the best girls in the class. (Correct)
22. The chair's legs are broken. (Incorrect)
The legs of the chair are broken. (Correct)
Note: In case of lifeless things, possession is shown by using the word 'of'.

23. The ship sank and all his passengers were drowned. (Incorrect)
The ship sank and all her passengers were drowned. (Correct)
Note: Some nouns are always used in the Feminine Gender; as, ship, country, etc.

24. The old man was knocking at the door of Death. (Incorrect)
The old man was knocking at Death's door. (Correct)
Note: Observe the use of possessive case with the followingDeath's door, duty's call, a
kilogram's weight.

25. He spent the last summer vacations with his uncle. (Incorrect)
He spent the last summer vacation with his uncle. (Correct)
26. She has two fifty-paisas coins. (Incorrect)
She has two fifty-paisa coins. (Correct)
27. All the family members were present. (Incorrect)
All the members of the family were present. (Correct)
28. Good night, Sir, I am glad to see you. (Incorrect)
Good evening, Sir, I am glad to see you. (Correct)
29. He has an urgent work. (Incorrect)
He has an urgent piece of work. (Correct)
30. Peculiar plurals of some words

(a) We should never desert our comrades and brothers. (Incorrect)


We should never desert our comrades and brethren. (Correct)
(b) Shakespeare, Milton and Dante are the three great genii who remain unsurpassed. (Incorrect)
Shakespeare, Milton and Dante are the three great geniuses who remain unsurpassed. (Correct)
31. I had no rupees. (Incorrect)
I had no money. (Correct)
Note: The word 'money' is used while talking about wealth. But we can say

I have a rupee.
I gave him two rupees.
32. He enjoyed the theatre. (Incorrect)
He enjoyed the play. (Correct)
Note: Theatre is a special building or place for the performance of plays. Thus, we enjoy the
play, not the theatre.

(a) I prefer gas cooks to electric ones. (Incorrect)


I prefer gas cookers to electric ones. (Correct)
(b) I saw a daughter on the road. (Incorrect)
I saw a girl on the road. (Correct)
33. (a) My husband is a very good cooker. (Incorrect)
(b) My husband is very good cook. (Correct)
Note: 'Cook' is a person who cooks food. 'Cooker' is an apparatus wherein food is cooked.

34. Don't trust him; he is a cheater. (Incorrect)


Don't trust him; he is a cheat. (Correct)
Note: The word 'cheat' is used in the British English whereas the word 'cheater' is used in
the American English. As the British English is flawless, the word 'cheat' is appropriate.

35. (a) My girl is too young to marry. (Incorrect)


(b) My daughter is too young to marry. (Correct)
Note: Girl is a female child whereas daughter is a female child in relation to her parents.

36. The weather of Ranchi suits me. (Incorrect)


The climate of Ranchi suits me. (Correct)
Note: Climate denotes the general atmospheric conditionsof a country whereas weather is
the conditions at a particular time.

37. This is the cause why I don't like it. (Incorrect)


This is the reason why I don't like it.(Correct)
or
This is why I don't like it.(Correct)
Note: 'Why' is never used after 'cause'. The word 'of' is used after 'cause'; as, 'What was
the cause of the accident?'

38. (a) He lost a quantity of books. (Incorrect)


He lost a number of books. (Correct)
(b) We had a large number of money. (Incorrect)
We had a large amount of money. (Correct)
Note: 'Quantity or amount' is used with an uncountable noun.

39. Sorry, Mr., I can't come. (Incorrect)


Sorry, Mr. Sen, I can't come. (Correct)
Sorry, Sir, I can't come.
Note: 'Mr. or Mrs.' is never used alone. It is always used with a name.

40. Do you like his new dress? (Incorrect)


Do you like his new clothes/suit? (Correct)
Do you like her new dress/clothes?
Note: 'Dresses' are generally worn by girls and ladies. The word 'clothes' means garments
such as trousers, dresses, shirts, etc. worn on the body.

41. (a) He got two breads. (Incorrect)


He got two loaves of bread. (Correct)
(b) I eat a bread daily. (Incorrect)
I eat bread daily. (Correct)
I eat a loaf of bread daily.
Note: 'Bread' is an uncountable noun. It is to be used with a numeral; always saya leaf/a
piece of bread.

42. (a) He applied for teachery. (Incorrect)


He applied for teachership. (Correct)
He applied for the post of teacher.
(b) I work for the upliftment of my village. (Incorrect)
I work for the uplift of my village. (Correct)
(c) He applied for lecturership. (Incorrect)
He applied for lectureship. (Correct)
(d) I got a freeship. (Incorrect)
I got a free studentship. (Correct)
Note: There is no word 'teachery' in the dictionary. 'Uplift' in itself is a noun. The suffix
ment is not added to it to make it a noun. 'Teacher' in itself is a noun. It is wrong to add ship
to it to make it a noun. Similarly, the correct word is free studentship, not 'freeship'.

43. (a) It is a nice poetry. (Incorrect)


It is a nice poem. (Correct)
(b) Quote the poem from heart. (Incorrect)
Quote the poem from memory. (Correct)
Note: 'Poetry' means the art of a poet or poems in general. It is an uncountable noun
whereas 'poem' is a countable noun. 'Get by heart' is an idiom which means 'learn
something'. But the thing which has been learnt is revised with the help of memory. So,
'quote from memory' is appropriate.

44. Please give me all informations. (Incorrect)


Please give me all information. (Correct)
Note: The word 'information' is an uncountable noun. It has no plural form. Neither is 'a' or
'an' used before it.

45. (a) I need some cardboards for this. (Incorrect)


I need some cardboard for this. (Correct)
(b) Our children need some educations. (Incorrect)
Our children need some education. (Correct)
(c) I need some new equipments. (Incorrect)
I need some new equipment. (Correct)
(d) I took some foods to eat. (Incorrect)
I took some food to eat. (Correct)
Note: 'Cardboard', 'education', 'equipment' and 'food' all are uncountable nouns. When
there is much food, we say a lot of food.

45. (a) We went there on horsebacks. (Incorrect)


We went there on horseback. (Correct)
(b) We heard some good musics. (Incorrect)
We heard some good music. (Correct)
(c) The news are bad today. (Correct)
The news is bad today. (Incorrect)
Note: 'Horseback', 'music' and 'news' all are uncountable nouns.

46. Two peoples were in the garden. (Incorrect)


Two people were in the garden. (Correct)
Note: 'People' is itself in plural. When it means a nation, it is treated singular. 's' is added to
it to make it plural; as, the peoples (= nations) of Europe

47. (a) In their free times, they played tennis. (Incorrect)


In their free time, they played tennis. (Correct)
(b) During their lesson times the children made a lot of noise. (Incorrect)
During their lesson time the children made a lot of noise. (Correct)
Note: 'free time' and 'lesson time' are uncountable. So, they can't be used in plural.

48. (a) They live in another towns. (Incorrect)


They live in another town. (Correct)
(b) Have you got other letter? (Incorrect)
Have you got other letters?
Have you got another letter?
Note: 'Another' is singular but towns is plural. A singular noun is used with 'another'. On the
other hand, 'other' is plural, so a plural noun (letters) is used with it.

49. (a) He hates all mother-in-laws (Incorrect)


He hates all mothers-in-law. (Correct)
(b) I have five brother-in-laws. (Incorrect)
I have five brothers-in-law. (Correct)
Note: The plurals of mother-in-law, father-in-law, brotherin- law, passer-by, etc. are mothers-
in-law, fathers-in-law, brothers-in-law, passers-by respectively.

50. (a) They are woman-teachers. (Incorrect)


They are women-teachers. (Correct)
(b) You are ladies doctors. (Incorrect)
You are lady-doctors. (Correct)
(c) You all are houses-maids. (Incorrect)
You all are house-maids. (Correct)
Note: The plural of a compound noun is got by making the plural of the second noun as, boy-
friendboy-friends, fountain-penfountain-pens and so on. In case a compound noun
contains man or woman as the first noun, its plural is got by making the plurals of both the
nouns; as, womenservant women-servants, women-doctorwomendoctors.

51. (a) We are Adam's and Eve's children. (Incorrect)


We are Adam and Eve's children. (Correct)
(b) Shakespeare and Tennyson's poetry. (Incorrect)
Shakespeare's and Tennyson's poetry. (Correct)
Note: When several words are in apposition, be careful to form their possessive with the 's
on the last word only if you are sure that they form a single sense unit and the result is free
from ambiguity.

52. (a) I am fond of Keat's poems. (Incorrect)


I am fond of Keats' poems. (Correct)
(b) Do you like Yeat's poetry? (Incorrect)
Do you like Yeats' poetry? (Correct)
Note: Only 's or ' is added to a singular proper noun which ends in 's'; as, Keats' poems or
Keats's poems, Hercules' labour or Hercules's labour and so on.

53. (a) Her house's number was not known.Incorrect)


The number of her house was not known. (Correct)
(b) The bell was of the front door's side. (Incorrect)
The bell was at the side of the front door. (Correct)
(c) It was a walk of five minutes. (Incorrect)
It was five minutes' walk. (Correct)
(d) I spent a holiday of a month. (Incorrect)
I spent a month's holiday. (Correct)
Note: 's is used with the animate; as, cow's milk, a women's college, a men's lavatory, my
father's name, the day's name, mother's hat, a child's toy, etc. But 's is used with the
inanimate, especially when related to time, distance and money; as, two days' journey, a
night's arrival, ten minutes' walk, government's decision, this year's sale, their money's
worth and so on.

54. His asset was seized. (Incorrect)


His assets were seized. (Correct)
Note: When the word 'asset' means a quality; it is written in the singular form; as, good
health is a great asset. Beauty is her only asset.

55. (a) Do you know the English alphabets? (Incorrect)


Do you know the English alphabet? (Correct)
(b) They are using new machineries. (Incorrect)
They are using new machinery. (Correct)
Note: 'Alphabet' and 'machinery' are not used in their plural forms.

56. (a) What a nonsense to have a picnic today! (Incorrect)


What nonsense to have a picnic today! (Correct)
(b) It is a nice weather. (Incorrect)
It is nice weather. (Correct)
(c) India is making a progress in the field of science. (Incorrect)
India is making progress in the field of science. (Correct)
Note: 'Nonsense', 'weather' and 'progress' are uncountable nouns. 'a' is not used before
them. But we can saya piece of nonsense, a spell of bad weather.

57. (a) By means of books knowledges are spread. (Incorrect)


By means of books knowledge is spread. (Correct)
(b) I saw a magic yesterday. (Incorrect)
I saw magic yesterday. (Correct)
Note: 'Knowledge' and magic' are uncountable nouns. 'a' is not used before them. Sometimes
'a' is used before knowledge; as,
He has a knowledge (= information about) of the truth.
He has a good knowledge (= information) of London.

58. (a) I like apple more than the orange. (Incorrect)


I like an apple more than an orange.(Correct)
or
I like apples more than oranges.(Correct)
(b) Ripe mango is sweeter than ripe guava. (Incorrect)
A ripe mango is sweeter than a ripe guava.(Correct)
or
Ripe mangoes are sweeter than ripe guavas.(Correct)
Note: The name of a fruit is used both in the singular form and in the plural form. So, 'a' or
'an' is used or it can be used in its plural form.

59. (a) I got the mark for the examination. (Incorrect)


I got the marks for the examination. (Correct)
(b) The teacher gave me full mark in mathematics. (Incorrect)
The teacher gave me full marks in mathematics. (Correct)
Note: The word 'marks' and not 'mark' is always used when a teacher gives them to a
student in the examination. 'Full marks' and not 'full mark' is the correct expression.

60. (a) They are called optimist. (Incorrect)


They are called optimists. (Correct)
(b) They are good actor. (Incorrect)
They are good actors. (Correct)
(c) We are a student. (Incorrect)
We are students. (Correct)
Note: 'They are' and 'we are' are plural expressions. They are followed by plural nouns.

61. (a) They could become a lawyer. (Incorrect)


They could become lawyers. (Correct)
(b) We were then a schoolboy. (Incorrect)
We were then schoolboys. (Correct)
(c) Children must become an honest man. (Incorrect)
Children must become honest men. (Correct)
Note: 'They', 'we' and 'children' are plural whereas 'a lawyer', 'a schoolboy' and 'a honest
man' are singular. How can so many persons become one particular man?

62. (a) They don't give much benefits. (Incorrect)


They don't give many benefits. (Correct)
(b) He has much children. (Incorrect)
He has many children. (Correct)
Note: 'Much' is used with an uncountable noun; as, much water, much milk, etc. 'Many' is
used with a countable noun; as, many boys, many pencils, etc.

63. (a) He went to the picture to see 'Gandhi'. (Incorrect)


He went to the pictures to see 'Gandhi'. (Correct)
(b) Our surrounding was pleasant. (Incorrect)
Our surroundings were pleasant. (Correct)
(c) We were in high spirit. (Incorrect)
We were in high spirits. (Correct)
Note: When the word 'pictures' means Cinema, it is always in plural. 'Surroundings' is always
in plural. 'High/Low/Good spirits' is always used in plural. Here, 'spirits' means 'mood'.

64. There are some who calls him a bore. (Incorrect)


There are some who call him a bore. (Correct)
Note: Here, 'some' is plural and 'who' points to 'some' which is the subject of 'call'. Thus
'call' has to be made plural.

65. (a) Nowaday the towns are very crowded. (Incorrect)


Nowadays the towns are very crowded. (Correct)
(b) Sometime I had milk for breakfast. (Incorrect)
Sometimes I had milk for breakfast. (Correct)
(c) His whereabout is still unknown. (Incorrect)
His whereabouts are still unknown. (Correct)
Note: 'Nowadays' is always used in plural. 'Sometimes' moment; as, see me some time
tomorrow. Similarly, 'whereabouts' is always used in plural.

66. (a) That pair of shoes are mine. (Incorrect)


That pair of shoes is mine. (Correct)
(b) This pair of scissors belong to me. (Incorrect)
This pair of scissors belongs to me. (Correct)
Note: 'A pair of shoes/shears/scissors/gloves/pants' is always treated singular. A singular
verb is used after it. The moment 'a pair of' is removed, it becomes plural and takes a plural
verb; as
, His shoes are dirty.
Those scissors belong to me.
His trousers were torn.
1. (a) One should love his country. (Incorrect)
One should love one's country. (Correct)
(b) One must do her work. (Incorrect)
One must do one's work. (Correct)
(c) One must do their duty. (Incorrect)
One must do one's duty. (Correct)
Note: If 'one' is the subject in a sentence, 'his', 'her', 'our', 'your' or 'their' is not used. Only
'one' is used with an 's.

2. (a) He had myself gone there. (Incorrect)


He had himself gone there. (Correct)
(b) Myself will do it. (Incorrect)
I will do it.(Correct)
or
I myself will do it.(Correct)
Note: Relative pronouns such as 'myself', 'yourself', 'itself', etc. are not used as subjects.

3. (a) I and you have done the work efficiently. (Incorrect)


You and I have done the work efficiently. (Correct)
(b) He and you went there. (Incorrect)
You and he went there. (Correct)
(c) I and he liked it. (Incorrect)
He and I liked it. (Correct)
(d) Tom, I and you ate sweets. (Incorrect)
You, Tom and I ate sweets. (Correct)
Note: Always keep in mind the 'Rule of 231' while using the pronouns of the first person,
second person, and the third person. This rule applies when something good has been done in
the sentence.

4. (a) He and I are at fault. (Incorrect)


I and he are at fault. (Correct)
(b) You, Dick and I have committed theft. (Incorrect)
I, you and Dick have committed theft. (Correct)
Note: Here, keep in mind the 'Rule of 123'. This rule applies when something bad has been
done in the sentence.

5. (a) These two boys help one another.


These two boys help each other. (Correct)
(b) These three boys help each other.
These three boys help one another. (Correct)
Note: 'Each other' is used when there are two persons or things whereas 'one another' is
used when there are three or more than three persons or things.

6. (a) Any of these two girls is your sister. (Incorrect)


Either of these two girls is your sister. (Correct)
(b) Either of the five boys will do it. (Incorrect)
Any of the five boys will do it. (Correct)
Note: 'Either of' is used for one of the two persons or things whereas 'any of' is used for
three or more than three persons or things.

7. (a) Neither of the six can succeed. (Incorrect)


None of the six can succeed. (Correct)
(b) None of the two girls is very tall. (Incorrect)
Neither of the two girls is very tell. (Correct)
Note: 'Neither of' means not any of the two whereas 'none of' is used for three or more than
three persons or things. It means 'not any of them'.

8. (a) Let you and I try. (Incorrect)


Let you and me try. (Correct)
(b) The principal wants to see you and I. (Incorrect)
The principal wants to see you and me. (Correct)
Note: The objective case of 'I' is 'me'.
9. (a) Tom is older than me. (Incorrect)
Tom is older than I. (Correct)
(b) Sita is taller than him. (Incorrect)
Sita is taller than he. (Correct)
(c) Lucy found more mushrooms than we did. (Incorrect)
Lucy found more mushrooms than I did. (Correct)
Note: In informed English, we often use object pronouns (we, him, her, us, them) after than.
In a more formal style, subject pronouns (I, He, etc.) are considered more correct.

10. (a) The climate of India is hotter than England. (Incorrect)


The climate of India is hotter than that of England.(Correct)
or
The climate of India is hotter than the climate of England.(Correct)
(b) The streets of Karachi are wider than Mumbai. (Incorrect)
The streets of Karachi are wider than those of Mumbai. (Correct)
Note: Here, the Indian climate has to be compared with the English climate, not with
England. Similarly, the streets of Karachi are to be compared with the streets of Mumbai, not
with Mumbai.

11. (a) Your's truly. (Incorrect)


Yours truly. (Correct)
(b) Your's is the best painting in the school. (Incorrect)
Yours (= your painting) is the best painting in the school. (Correct)
Note: 'Yours' is never written as 'your's'.

12. (a) Your pen is better than that of mine. (Incorrect)


Your pen is better than mine. (Correct)
(b) My book is cheaper than that of yours. (Incorrect)
My book is cheaper than yours. (Correct)
Note: Here, 'mine' means 'my pen' and 'yours' means 'your book'. So, 'that of mine/yours is
absolutely incorrect'.

13. (a) He absented from the class. (Incorrect)


He absented himself from the class. (Correct)
(b) I will avail of this opportunity. (Incorrect)
I will avail myself of this opportunity. (Correct)
(c) Ramesh acquitted well in the debate. (Incorrect)
Ramesh acquitted himself well in the debate. (Correct)
(d) Smart people sometimes over-reach. (Incorrect)
Smart people sometimes over-reach themselves. (Correct)
(e) I will avenge on my enemies. (Incorrect)
I will avenge myself on my enemies. (Correct)
(f) You would have passed your exams if you had applied. (Incorrect)
You had applied yourself. (Correct)
(g) In order to be successful he would have to exert. (Incorrect)
In order to be successful he would have to exert himself. (Correct)
(h) After leaving college, he set up as a freelance photographer. (Incorrect)
After leaving college, he set himself up as a freelance photographer. (Correct)
(i) Let us enjoy. (Incorrect)
Let us enjoy ourselves. (Correct)
Note: The following verbs are generally followed by the reflexive pronouns; as, absent,
acquit, avail, avenge, apply, exert, over-reach, set, enjoy.

14. (a) He qualified himself as a teacher. (Incorrect)


He qualified as a teacher. (Correct)
(b) We will hide ourselves in the garden. (Incorrect)
We will hide in the garden. (Correct)
(c) They begin to make themselves merry. (Incorrect)
They begin to make merry. (Correct)
(d) Have you enlisted yourself in the army? (Incorrect)
Have you enlisted in the army? (Correct)
(e) He kept himself away from the meeting. (Incorrect)
He kept away from the meeting. (Correct)
(f) He bathes himself every morning. (Incorrect)
He bathes every morning. (Correct)
(g) He finally managed to break himself free from his attacker. (Incorrect)
He finally managed to break free from his attacker. (Correct)
(h) He dashed himself off to keep an appointment. (Incorrect)
He dashed off to keep an appointment. (Correct)
(i) The politician moved himself quickly to dispel the rumours. (Incorrect)
The politician moved quickly to dispel the rumours. (Correct)
(j) He drew himself a circle in the sand with a stick. (Incorrect)
He drew a circle in the sand with a stick. (Correct)
(k) He won't rest himself until he finds her. (Incorrect)
He won't rest until he finds her. (Correct)
(l) The ball rolled itself down the hill. (Incorrect)
The ball rolled down the hill. (Correct)
(m) He burst himself into the room without knocking. (Incorrect)
He burst into the room without knocking. (Correct)
(n) They have a large family to feed themselves. (Incorrect)
They have a large family to feed. (Correct)
Note: The following verbs are not followed by reflexive pronouns; as, break, bathe, dash,
keep, move, make, spread, draw, rest, roll, enlist, burst, hide, feed, qualify.

15. (a) Every teacher and every student should do their duty. (Incorrect)
Every teacher and every student should do his duty. (Correct)
Note: When two singular nouns joined by 'and' are preceded by each or every, the pronoun
will be singular.

16. (a) Either you or he should do their duty. (Incorrect)


Either you or he should do his duty. (Correct)
(b) Either the master or his servants failed to do his duty. (Incorrect)
Either the master or his servants failed to do their duty. (Correct)
Note: The pronoun is singular when two singular noun are joined by 'either-or' or 'neither-
nor'. The pronoun comes according to the noun nearest to the verb. The pronoun is plural
when a singular noun and a plural noun are joined. But in such cases, the singular noun is
placed first.

17. Nobody can accomplish this task but I. (Incorrect)


Nobody can accomplish this task but me. (Correct)
Note: When a pronoun is used as the object of a verb or preposition, it must be in the
objective form.

18. It is me. (Incorrect)


It is I. (Correct)
Note: The expression 'It is me' is the American English. When a pronoun is the complement
of the verb 'to be', it must be in the nominative case.

19. It is I who is to blame. (Incorrect)


It is I who am to blame. (Correct)
Note: The relative pronoun always agrees with its antecedent in number, gender and person.

20. (a) Only those students should apply for the post who have passed their B.A. Examination.
(Incorrect)
Only those students who have passed the B.A. Examination should apply for the post. (Correct)
(b) This is Rama's house who is a fast friend of mine. (Incorrect)
This is the house of Rama who is a fast friend of mine. (Correct)
Note: A noun or pronoun in the possessive case should not be used as the antecedent of a
relative pronoun.

21. The child who fell into the river was saved. (Incorrect)
The child that fell into the river was saved. (Correct)
Note: 'That' should be used after a noun whose gender is doubtful.

22. No student who shirks work can get a good position. (Incorrect)
No student that shirks work can get a good position. (Correct)
Note: 'That' should be used after the negative.

23. Such students who burn the mid-night oil are always successful. (Incorrect)
Such students as burn the mid-night oil are always successful. (Correct)
Note: 'As' as a relative pronoun is used after 'such' or 'same'.

24. (a) There was none but did not weep. (Incorrect)
There was none but wept. (Correct)
(b) There was none but did not laugh. (Incorrect)
There was none but laughed. (Correct)
Note: 'But' as a relative pronoun means 'who did not' or 'which did not'.

25. Who is more intelligent, Shyam or Sunder? (Incorrect)


Which is more intelligent, Shyam or Sunder? (Correct)
Note: 'Which' is used for selection even with persons.
26. (a) Whom, do you think, has stolen the book? (Incorrect)
Who, do you think, has stolen the book? (Correct)
(b) This is Sachin Tendulkar whom, they say is the best cricketer in India. (Incorrect)
This is Sachin Tendulkar who, they say, is the best cricketer in India. (Correct)
Note: 'Who' is used in the nominative case whereas 'whom' is used in the objective case.

27. Who are you speaking to? (Incorrect)


Whom are you speaking to? (Correct)
Note: A pronoun governed by a preposition must be in the objective case.

28. I object to you saying that. (Incorrect)


I object to your saying that. (Correct)
29. My ideas are different from you. (Incorrect)
My ideas are different from yours. (Correct)
30. Now I take your leave. (Incorrect)
Now I take leave of you. (Correct)
31. Whose lives near you house? (Incorrect)
Who lives near your house? (Correct)
32. The jury were divided in its opinion. (Incorrect)
The jury were divided in their opinions. (Correct)
33. All which glitters is not gold. (Incorrect)
All that glitters is not gold. (Correct)
34. He made your mention. (Incorrect)
He made a mention of you. (Correct)
1. He dead. (Incorrect)
He has died.
He died.
He was dead. (Correct)
He is dead.
He had died.
Note: 'Dead' is an adjective whereas 'died' is the past tense and past participle of the verb
'to die'.

2. He gave me little money to spend. (Incorrect)


He gave me a little money to spend. (Correct)
Note: 'Little' means nothing whereas 'a little' means something.

3. (a) The book is more costly than the other. (Incorrect)


The book is costlier than the other. (Correct)
(b) This is heavy of all the stones. (Incorrect)
This is the heaviest of all the stones. (Correct)
(c) I am the younger in the family. (Incorrect)
I am the youngest in the family. (Correct)
(d) Your account is worst than mine. (Incorrect)
Your account is worse than mine. (Correct)
(e) This one is worst than that one. (Incorrect)
This one is worse than that one. (Correct)
(f) Which is the better of the three methods? (Incorrect)
Which of the three methods is the best? (Correct)
Note: The comparative degree is used when we compare two things and the superlative
degree is used when there are more than two persons or things.

4. (a) I have a toothache, so I can't eat something. (Incorrect)


I have a toothache, so I can't eat anything. (Correct)
(b) I have any books. (Incorrect)
I have some books. (Correct)
(c) He hasn't some pens. (Incorrect)
He hasn't any pens. (Correct)
(d) Have you some sugar? (Incorrect)
Have you any sugar? (Correct)
Note: 'Some' is generally used in affirmative or declarative sentences whereas 'any' is used
in negative sentences and interrogative ones.

5. (a) The weather is warmer as the last week. (Incorrect)


The weather is warmer than that of the last week. (Correct)
(b) Rekha is taller from her sister. (Incorrect)
Rekha is taller than her sister. (Correct)
Note: 'Than' is used in the comparative degree.

6. (a) Iron is more useful than any metal. (Incorrect)


Iron is more useful than any other metal. (Correct)
(b) The USA is the richest of all other countries. (Incorrect)
The USA is the richest of all the countries. (Correct)
(c) He was the most courteous and the noblest person in the town. (Incorrect)
He was the noblest and most courteous person in the town. (Correct)
Note: When the adjectives in the comparative degrees or superlative degrees come together,
the one formed by adding 'more' or 'most' should follow the other adjective.

7. We should help the poors. (Incorrect)


We should help the poor. (Correct)
Note: Adjectives preceded by the article 'the' and used as plural common nouns do not take
's' after them.

8. (a) My boy-friend is very older than me. (Incorrect)


My boy-friend is much older than I am. (Correct)
(b) You are far much tolerant than I am. (Incorrect)
You are far more tolerant than I am. (Correct)
Note: 'Very' is never used with the comparative degree. 'Much' or 'far' is used instead. But
'very much', 'a lot', 'lots', 'any', 'no', 'rather', 'a little', 'a bit', etc. are a few words which are
used with the comparative degree; as,

Very much richer


a lot happier
rather more quickly
a little less expensive
a bit easier
Things are no better than before.
9. (a) This is a more safer place. (Incorrect)
This is a safer place. (Correct)
(b) The most busiest hour is 10 o'clock in the morning. (Incorrect)
The busiest hour is 10 o'clock in the morning. (Correct)
Note: A double comparative (= more safer) or a double superlative (= most busiest) is never
used.

10. (a) This photograph is the best of the two. (Incorrect)


This photograph is the better of the two. (Correct)
(b) Tom is the tallest of the two boys. (Incorrect)
Tom is the taller of the two boys. (Correct)
Note: A comparative degree is used where two persons or things are mentioned.

11. This little girl is better than clever. (Incorrect)


This little girl is more good than clever. (Correct)
Note: 'More' is used where two qualities of the same person/thing are compared.
12. Open your book at twenty-fifth page. (Incorrect)
Open your book at page twenty-five. (Correct)
13. Bipan was elder than Rama. (Incorrect)
Bipan was older than Rama. (Correct)
Note: The word 'elder' is never followed by 'than'. Always sayShe is my elder sister; he is
my elder brother.

14. These all apples have become rotten. (Incorrect)


All these apples have become rotten. (Correct)
15. From the two he is clever. (Incorrect)
He is the cleverer of the two. (Correct)
16. He lives in the most excellent house. (Incorrect)
He lives in an excellent house. (Correct)
Note: Some adjectives being superlative in meaning do not admit of comparison, as,
excellent, entire, extreme, ideal, perfect, unique, universal, etc.

17. He is inferior than me. (Incorrect)


He is inferior to me. (Correct)
Note: The adjectives ending in 'or' are followed by 'to', not by 'than'. These adjectives are
eight in number; as, inferior, superior, junior, senior, anterior, posterior, ulterior, prior.

18. Of the threeorange, apple and mangothe latter is my favourite fruit. (Incorrect)
Of the threeorange, apple and mangothe last is my favourite fruit. (Correct)
Note: Use 'latter' for two and 'last' for more than two.

19. (a) Ambala is further than Karnal from Delhi. (Incorrect)


Ambala is farther than Karnal from Delhi. (Correct)
(b) America is ready to give us farther help. (Incorrect)
America is ready to give us further help. (Correct)
Note: 'Farther' means more distant whereas 'further' means additional.

20. (a) His oldest son is a lawyer. (Incorrect)


His eldest son is a lawyer. (Correct)
(b) Lata is the older of the two sisters. (Incorrect)
Lata is the elder of the two sisters. (Correct)
Note: 'Elder' and 'eldest' are used only for persons belonging to the same family. 'Older' and
'oldest' are used for both persons and things.

21. No less than three students were fired. (Incorrect)


No fewer than three students were fired. (Correct)
Note: 'Less' refers to quantity whereas 'fewer' denotes number.

22. (a) I saw a most unique sight. (Incorrect)


I saw a unique sight. (Correct)
(b) Is there anything more eternal? (Incorrect)
Is there anything eternal? (Correct)
(c) I found that dog more dead. (Incorrect)
I found that dog dead. (Correct)
Note: Adjectives like unique, eternal, round, dead, triangular, golden, entire, chief, dumb,
free, blind, universal, ideal, weekly, monthly, etc. are never used with comparative degrees
and superlative degrees.

Explanation:According to F.T. Wood, the world-famous English grammarian, adjectives like


unique, dead, blind, dumb, which express an absolute idea of which there can't be degrees,
obviously can't have a comparative degree. One thing can't be more unique than another or one
person more dead or more dumb than another.
23. (a) My brother has not much book. (Incorrect)
My brother has not many books. (Correct)
(b) Is there many sugar in the bottle? (Incorrect)
Is there much sugar in the bottle? (Correct)
Note: 'Much' refers to quantity whereas 'many' refers to number.

24. (a) He is the tallest boy of the school. (Incorrect)


He is the tallest boy in the school. (Correct)
(b) It is the most expensive car of the world. (Incorrect
) It is the most expensive car in the world. (Correct)
Note: After superlative degrees, we use in (or other prepositions) to show what place we are
talking about; as,
It is the most expensive car in the world.
I am the happiest girl under the sun.
'of' is not normally used, but it is possible after a superlative degree without a noun; as,
She is the most sensible of the Smith Girls. 25. (a) It is best book I have ever read. (Incorrect)

It is the best book I have ever read. (Correct)


(b) He is fastest runner here. (Incorrect)
He is the fastest runner here. (Correct)
Note: 'The' is added before a superlative degree.

But there are a few exceptions to this rule


(a) That is most (= very) kind of you.
(b) Which do you like best?
(c) I like that one most.
(d) I buy the pen that costs least.
Note: When 'most' is used as 'very', 'the' is omitted. Here 'like best' or 'cost least' have
become set phrases which don't need 'the' before them. Sometimes superlative adjectives
are used without nouns; as, I am the greatest; this one is the fastest.

26. (a) I saw an asleep man. (Incorrect)


I saw a man asleep. (Correct)
(b) An alone lady might be in danger here. (Incorrect)
A lady alone might be in danger here. (Correct)
(c) That is an ill man. (Incorrect)
That man is ill. (Correct)
(d) These two all alike friends. (Incorrect)
These two are friends alike. (Correct)
Note: There are some adjectives like asleep, alone, ill, afraid, akin, alike, awake, ashamed,
averse, etc. which are used after nouns.

27. It was done in a remarkable clever manner. (Incorrect)


It was done in a remarkably clever manner. (Correct)
Note: 'Remarkable' is an adjective; it can't qualify another adjective, i.e. 'clever'.

28. That was a worth-seeing spectacle. (Incorrect)


That was a spectacle worth seeing. (Correct)
Note: 'Worth-seeing' is not a one-word adjective.

29. I have leave of three days only. (Incorrect)


I have three days' leave only. (Correct)
I have leave for three days only.
30. I have never seen a so good servant. (Incorrect)
I have never seen so good a servant. (Correct)
31. He was fined hundred rupees. (Incorrect)
He was fined a hundred rupees. (Correct)
32. He took it up with his both hands. (Incorrect)
He took it up with both his hands. (Correct)
33. (a) Many a men hanker after wealth. (Incorrect)
Many a man hankers after wealth. (Correct)
(b) His illness was very heavy. (Incorrect)
His illness was very serious. (Correct)
(c) They had a heavy quarrel. (Incorrect)
They had a serious quarrel. (Correct)
(d) I know him very good. (Incorrect)
I know him very well. (Correct)
Note: 'Many a' is followed by a singular noun and a singular verb. 'Heavy' denotes weight
but 'serious' means important because of possible danger; as, a serious mistake, a serious
crime.

34. (a) I am two years smaller than you. (Incorrect)


I am two years younger than you. (Correct)
(b) He is three years bigger than you. (Incorrect)
He is three years older than you. (Correct)
Note: 'Smaller' and 'bigger' denote size whereas 'younger' and 'older' denote age.

35. (a) He has grown into a beautiful youth. (Incorrect)


He has grown into a handsome youth. (Correct)
(b) She has grown into a handsome young girl. (Incorrect)
She has grown into a beautiful young girl. (Correct)
Note: For beauty, 'beautiful' is used for woman and 'handsome' is used for man.

36. Everyone had left him; he had a few friends now. (Incorrect)
Everyone had left him; he had few friends now. (Correct)
Note: 'A few' means some 'few' means none. The content of the sentence implies that he had
no friends, therefore 'few' and not 'a few' is used.

37. It is happy to note that our teams has won the match. (Incorrect)
It is gratifying to note that our team has won the match. (Correct)
38. Whole town came to receive him. (Incorrect)
The whole town (= the whole of town) came to receive him. (Correct)
39. How happy we are at home than in college! (Incorrect)
How much happier we are at home than in college! (Correct)
40. We found his house with somewhat difficulty. (Incorrect)
We found his house with some difficulty. (Correct)
Note: 'Somewhat' is an adverb and cannot qualify a noun.

41. You only are responsible for this tragedy. (Incorrect)


You alone are responsible for this tragedy. (Correct)
42. (a) This is my every day clothing. (Incorrect)
This is my everyday clothing. (Correct)
(b) He goes to school everyday. (Incorrect)
He goes to school every day. (Correct)
Note: When 'everyday' is used as an adjective, i.e. when it is followed by a noun, it is one
word, otherwise it is written in two words, i.e. every day.

43. (a) He lives nearby. (Incorrect)


He lives near by. (Correct)
(b) My uncle put up in a near by hotel. (Incorrect)
My uncle put up in a nearby hotel. (Correct)
Note: when 'nearby' is used as an adjective, it is one word, otherwise it is written in two
words, i.e. near by.

44. (a) He is a greedy man for money. (Incorrect)


He is a man greedy for money. (Correct)
(b) All these are worthy matters of attention. (Incorrect)
All these are matters worthy of attention. (Correct)
Note: An adjective can be placed after a noun when the adjective is followed by a
prepositional phrase.

45. The two first chapters of this book are interesting. (Incorrect)
The first two chapters of this book are interesting. (Correct)
Note: 'The two first' is a meaningless expression, so 'the first two' should be used.

46. (a) He acted well in comedies than in tragedies. (Incorrect)


He acted better in comedies than in tragedies. (Correct)
(b) My English knowledge is sound. (Incorrect)
My knowledge of English is sound. (Correct)
(c) He is my family member. (Incorrect)
He is a member of my family. (Correct)
Note: Comparative degree is 'better', and not 'well'; 'knowledge of English' is a correct
expression. The correct expression is 'member of my family'.

47. (a) Older I get, happier I am. (Incorrect)


The older I get, the happier I am. (Correct)
(b) More you work, less you learn. (Incorrect)
The more you work, the less you learn. (Correct)
(c) The more it is dangerous, the more I like it. (Incorrect)
The more dangerous it is, the more I like it. (Correct)
Note: I special constructions, thethe is used with the comparative degrees to state that two
changes happen together. Never separate an adjective, an adverb or a noun from 'more' [see
sentence (c) below].

48. From there I went to the next hotel. (Incorrect)


From there, I went to the nearest hotel. (Correct)
Note: 'Nearest' denotes distance and 'next' denotes position.

49. What is the last news? Incorrect)


What is the latest news? (Correct)
Note: 'Latest' means the most recent whereas 'last' denotes order.

50. He has secured passing marks. (Incorrect)


He has secured pass marks. (Correct)
51. You are the nicest in the lot. (Incorrect)
You are the nicest of the lot. (Correct)
Note: 'Of-structure' is generally used as a superlative adjective.

52. (a) It seems to me a good and fine job. (Incorrect)


It seems to me a better and finer job. (Correct)
(b) He is taller and old than my friend. (Incorrect)
He is taller and older than my friend. (Correct)
(c) I got the sweetest and charming thing. (Incorrect)
I got the sweetest and most charming thing. (Correct)
Note: If a sentence contains two adjectives in different degrees, the degree in the second
part comes according to the degree in the first part.

53. (a) Health is more preferable than wealth. (Incorrect)


Health is preferable to wealth. (Correct)
(b) Knowledge is even more preferable than riches. (Incorrect)
Knowledge is preferable even to riches. (Correct)
(c) We prefer reading than writing. (Incorrect)
We prefer reading to writing. (Correct)
We prefer to read rather than to write.
Note: 'Preferable' is never followed by than. It means more desirable. Similarly, 'Prefer' is
always followed by 'to', not by 'than'.

54. (a) He was the laziest of all other workmen. (Incorrect)


He was the laziest of all workmen. (Correct)
(b) She was the best of all other singers. (Incorrect)
She was the best of all singers. (Correct)
Note: One should never use 'other' after a superlative degree of adjective.

55. (a) M.K. Gandhi was a greater leader of India. (Incorrect)


M.K. Gandhi was a great leader of India. (Correct)
(b) He is a more intelligent student of the class. (Incorrect)
He is a very intelligent student of the class. (Correct)
Note: If comparison is not crystal clear, no comparative degree is used.

56. This house is elder than that house. (Incorrect)


This house is older than that house. (Correct)
Note: For comparing things, 'older', and not 'elder', is used.

57. (a) There is no any boy in the class. (Incorrect)


There is no boy in the class. (Correct)
(b) He has no a hat. (Incorrect)
He has no hat. (Correct)
(c) He has not a book. (Incorrect)
He has no book. (Correct)
Note: 'Not + a' equals 'no' and 'not any' and 'not a' also equal 'no'.

58. (a) He is one of the taller boys in the class. (Incorrect)


He is one of the tallest boys in the class. (Correct)
(b) John is one of the more intelligent boys I know. (Incorrect)
John is one of the most intelligent boys I know. (Correct)
Note: The adjective followed by 'one of' always comes in the superlative degree. 59. (a) I am
very fond of these kind of flowers. (Incorrect)

I am very fond of this kind of flowers.(Correct)


or
I am very fond of the flowers of this kind.(Correct)
(b) I never like those sort of people. (Incorrect)
I never like that sort of people.(Correct)
or
I never like people of that sort.(Correct)
Note: The plurals of 'this' and 'that' are these and 'those' respectively. If the noun is
singular, a singular adjective is used for it. 'Kind' and 'sort' are singular nouns, hence singular
adjectives are used for them.
Order for placing various adjectives

Sometimes several adjectives qualify a single noun. There is no fixed rule as to how the order for
placing various adjectives should be. But the following table, clarifies the point.
From the above table, we conclude:
A high brick wall. (Correct)
A brick high wall. (Incorrect)
An old gold watch. (Correct)
A gold old watch. (Incorrect)
A valuable brown leather belt. (Correct)
A leather brown valuable belt. (Incorrect)
1. I am very much sorry to disappoint you. (Incorrect)
I am very sorry to disappoint you. (Correct)
2. This novel is too amusing. (Incorrect)
This novel is very amusing. (Correct)
3. (a) Everybody feels contentedly. (Incorrect)
Everybody feels contented. (Correct)
(b) The music sounds pleasantly. (Incorrect)
The music sounds pleasant. (Correct)
(c) That milk smells sourly. (Incorrect)
That milk smells sour. (Correct)
(d) This food tastes sweetly. (Incorrect)
This food tastes sweet. (Correct)
(e) She looks sadly. (Incorrect)
She looks sad. (Correct)
(f) He seems happily. (Incorrect)
He seems happy. (Correct)
Note: If the subject is followed by words like feel, seem, look, appear, sound, taste or smell,
the word following any of these adverbs can be used in the following cases:-
He looked at us sadly/happily/angrily.
She tasted/smelt the food carefully/quickly.
They sounded the bell loudly/noisily.

4. (a) I don't hardly know this girl. (Incorrect)


I hardly know this girl. (Correct)
(b) He can't scarcely see in this light. (Incorrect)
He can scarcely see in this light. (Correct)
Note: The words 'hardly and scarcely' mean probably. The use of 'not' with them is not
allowed as they convey negative sense.

5. He was too glad to see his uncle. (Incorrect)


He was very glad to see his uncle. (Correct)
6. I lived there seven years before. (Incorrect)
I lived there seven years ago. (Correct)
7. The deer ran fastly. (Incorrect)
The deer ran fast. (Correct)
8. I am very obliged to you. (Incorrect)
I am much obliged to you. (Correct)
9. The Principal was much angry with me. (Incorrect)
The Principal was very angry with me. (Correct)
10. He is somewhat tall for his age. (Incorrect)
He is rather tall for his age. (Correct)
11. He is comparatively better today. (Incorrect)
He is comparatively good today.(Correct)
or
He is better today.(Correct)
12. (a) I don't know to dance. (Incorrect)
I don't know how to dance. (Correct)
(b) I don't know to ride a bicycle. (Incorrect)
I don't know how to ride a bicycle. (Correct)
(c) I know to make omelettes. (Incorrect)
I know how to make omelettes. (Correct)
Note: 'Know' is never followed by any infinitive (= to dance, to ride, to make) formation;
always say or writeknow how to.

13. (a) He has thirty-five. (Incorrect)


He is thirty-five. (Correct)
(b) He is thirty-five years. (Incorrect)
He is thirty-five old.
He is thirty-five years old. (Correct)
Note: One can tell his/her age by just giving the number or by giving the number plus the
expression 'years old'. You had better askHow old are you? instead of askingwhat is
your age?

14. (a) I hit him hardly. (Incorrect)


I hit him hard. (Correct)
(b) He was hardly hit by his friend's death. (Incorrect)
He was hard hit by his friend's death. (Correct)
Note: 'Hardly' means probably not whereas 'hard' means forcefully.

15. You behaved cowardly. (Incorrect)


You behaved like a coward.(Correct)
or
You behaved in a cowardly manner.(Correct)
16. Exercise is quite useful for health. (Incorrect)
Exercise is very useful for health. (Correct)
17. The train will arrive just now. (Incorrect)
The train will arrive soon. (Correct)
18. Please kindly do it for me. (Incorrect)
Please do it for me.(Correct)
or
Kindly do it for me.(Correct)
19. He told the whole thing in details. (Incorrect)
He told the whole thing in detail. (Correct)
20. She is feeling very well today. (Incorrect)
She is feeling quite well today. (Correct)
21. The flower smells sweetly. (Incorrect)
The flower smells sweet. (Correct)
22. It was bitter cold. (Incorrect)
It was bitterly cold. (Correct)
23. (a) They came always late to school. (Incorrect)
They always came late to school. (Correct)
(b) He will be tomorrow here. (Incorrect)
He will be here tomorrow. (Correct)
(c) Come at four o'clock to my office. (Incorrect)
Come to my office at four o'clock. (Correct)
(d) I went at once there. (Incorrect)
I went there at once. (Correct)
Note: The rule of end-position adverbs is as follows adverb of manner (how) + adverb of
place (where) + adverb of time (when); as she sang perfectly/in the town hall/last night (=
manner + place + time)

24. (a) He came always late.


He always came late.
(b) She did it never.
She never did it.
(c) We went often there.
We often went there.
(d) I visit him while in Delhi.
I frequently visit him while in Delhi.
Note: The adverbsalways, never, often, generally, sometimes, quite, hardly, nearly,
frequently, almost, just, usually and scarcelycome between the subjects and the principal
verbs.

25. (a) He always is early. (Incorrect)


He is always early. (Correct)
(b) She never has been early. (Incorrect)
She has never been early. (Correct)
(c) They often will go there. (Incorrect)
They will often go there. (Correct)
Note: The adverbsalways, never, often, generally, sometimes, often, usually, already,
almost, just, nearly, quite, hardly, scarcelyare used after verb 'to be' (is, am, are) and
the auxiliaries.

26. (a) The water of this river is too pure. (Incorrect)


The water of this river is very pure. (Correct)
(b) He is very blind to read small print. (Incorrect)
He is too blind to read small print. (Correct)
(c) This house is too much small for me. (Incorrect)
This house is very small for me.(Correct)
or
This house is much too small for me.(Correct)
Note: The adverb 'too' means 'more than enough' and should not be used instead of 'very or
much'.

27. (a) I am quite sorry to hear it. (Incorrect)


I am very sorry to hear it. (Correct)
(b) This climate is quite injurious to health. (Incorrect)
This climate is very injurious to health. (Correct)
(c) He is almost quite well. (Incorrect)
He is quite well. (Correct)
Note: Quite means 'perfectly or completely' whereas very means 'much'. No qualifying word
(like 'almost') is used before the word 'quite'.

28. He is very friendly enough to help me. (Incorrect)


He is friendly enough to help me. (Correct)
Note: 'Enough' means 'in the proper limit or amount'. So, the use of 'very' or 'much' before
it is incorrect.

29. I request you to kindly grant me leave for three days. (Incorrect)
I request you kindly to grant me leave for three days. (Correct)
30. She was named as Lata. (Incorrect)
She was named Lata. (Correct)
31. Students entered the hall by and by. (Incorrect)
Students entered the hall one by one. (Correct)
32. Mahatma Gandhi has often been called as his country's saviour. (Incorrect)
Mahatma Gandhi has often been called his country's saviour. (Correct)
33. (a) He eagerly wishes to do it. (Incorrect)
He wishes eagerly to do it. (Correct)
(b) She sang last night beautifully. (Incorrect)
She sang beautifully last night. (Correct)
Note: An adverb is placed after the word it qualifies.

34. (a) Nobody saw us there fortunately. (Incorrect)


Fortunately, nobody saw us there. (Correct)
(b) He was fortunately not present there. (Incorrect)
Fortunately, he was not present there. (Correct)
Note: If an adverb qualifies the whole sentence, it is placed in the beginning of the sentence.

35. (a) He quickly ran. (Incorrect)


He ran quickly. (Correct)
(b) They immediately disappeared. (Incorrect)
They disappeared immediately. (Correct)
Note: If an adverb qualifies an intransitive verb, in that case the adverb comes after the
intransitive verb.

36. (a) I only worked out two sums. (Incorrect)


I worked out only two sums. (Correct)
(b) He only succeeded in the exam. (Incorrect)
Only he succeeded in the exam. (Correct)
Note: The adverb 'only' should be placed just before the word it qualifies.

37. (a) No boy other but Hari has done it. (Incorrect)
No boy other than Hari has done it. (Correct)
(b) It is nothing else than pride. (Incorrect)
It is nothing else but pride. (Correct)
Note: 'No other' is followed by 'than' whereas 'else' is followed by 'but' not by 'than'.
38. We returned more quicker than we expected. (Incorrect)
We returned more quickly than we expected. (Correct)
Note: Never use on adjective in place of an adverb.

39. (a) I seldom or ever refuse a request. (Correct)


I seldom or never refuse a request. (Incorrect)
(b) I seldom or ever find a mistake in this book. (Incorrect)
I seldom or never find a mistake in this book. (Correct)
Note: 'Seldom or never' or 'seldom, ever,' is the correct expression. But 'seldom or ever' is
an incorrect expression.

40. (a) I have not got no paper for my exercise. (Incorrect)


I have not got any paper for my exercise. (Correct)
(b) I forbade you not to enter the room. (Incorrect)
I forbade you to enter this room. (Correct)
(c) That will not stop him; nothing never did. (Incorrect)
That will not stop him; nothing ever did. (Correct)
(d) Unless you don't labour hard, you can't succeed. (Incorrect)
Unless you labour hard, you can't succeed. (Correct)
Note: The use of double negative makes a sentence an affirmative one. So, avoid the use of
double negative in the sentence.
1. He lives in Karol Bagh at Delhi. (Incorrect)
He lives at Karol Bagh in Delhi. (Correct)
2. (a) I have not seen him from a long time. (Incorrect)
I have not seen him for a long time. (Correct)
(b) She has been writing a letter for one o'clock. (Incorrect)
She has been writing a letter since one o'clock. (Correct)
Note: 'For' is used for period of time; as, for two days, for five months, etc. 'Since' is used
for point of time; as, since Monday, since 2 o'clock, etc.

3. He jumped in the well. (Incorrect)


He jumped into the well. (Correct)
4. A Punjabi differs with a Bengali. (Incorrect)
A Punjabi differs from a Bengali. (Correct)
5. Listen me. (Incorrect)
Listen to me. (Correct)
6. She burst in tears. (Incorrect)
She burst into tears. (Correct)
7. He took leave from his parents. (Incorrect)
He took leave of his parents. (Correct)
8. He knocked the door. (Incorrect)
He knocked at the door. (Correct)
9. The police caught him from his neck. (Incorrect)
The police caught him by the neck. (Correct)
10. (a) He fell in love of her. (Incorrect)
He fell in love with her. (Correct)
(b) The garden was covered from flowers. (Incorrect)
The garden was covered with flowers. (Correct)
(c) Compare this bag to a really good one. (Incorrect)
Compare this bag with a really good one. (Correct)
(d) The poet compared her lips with roses. (Incorrect)
The poet compared her lips to roses. (Correct)
Note: Use 'compare with' while comparing two persons or things of equal rank. Use 'compare
to' while comparing two things or persons of unequal rank.

11. I have no house to live. (Incorrect)


I have no house to live in. (Correct)
12. We should obey to our teachers. (Incorrect)
We should obey our teachers. (Correct)
13. He got down the train. (Incorrect)
He got down from the train.(Correct)
or
He got off the train.(Correct)
14. He has oil in hair. (Incorrect)
He has oil on hair. (Correct)
15. I am a teacher in English. (Incorrect)
I am a teacher of English. (Correct)
16. I am tired from this work. (Incorrect)
I am tired of this work. (Correct)
17. He came to school with bicycle. (Incorrect)
He came to school by bicycle. (Correct)
18. I don't care of this. (Incorrect)
I don't care for this. (Correct)
19. I can see two boys on the picture. (Incorrect)
I can see two boys in the picture. (Correct)
20. I listened to the music by the radio. (Incorrect)
I listened to the music on the radio. (Correct)
21. Who is responsible of this? (Incorrect)
Who is responsible for this? (Correct)
22. He reminded me to this. (Incorrect)
He reminded me of this. (Correct)
23. She suffers at fever. (Incorrect)
She suffers from fever. (Correct)
24. Everybody is surprised on our claim. (Incorrect)
Everybody is surprised at our claim. (Correct)
25. What is wrong to Hari? (Incorrect)
What is wrong with Hari? (Correct)
26. (a) Stop throwing stones to the cars. (Incorrect)
Stop throwing stones at the cars. (Correct)
(b) Throw the ball straight at the wicket-keeper. (Incorrect)
Throw the ball straight to the wicket-keeper. (Correct)
Note: Throw + object + at (aggressive); throw + object + to (in a game, etc.)

27. (a) Don't discuss about your plan. (Incorrect)


Don't discuss your plan. (Correct)
(b) He describes about Nature. (Incorrect)
He describes Nature. (Correct)
Note: No preposition is used after 'discuss' or 'describe'.

28. (a) The glass is full with water. (Incorrect)


The glass is full of water. (Correct)
(b) Are you satisfied from her? (Incorrect)
Are you satisfied with her? (Correct)
(c) My pen is superior from yours. (Incorrect)
My pen is superior to yours. (Correct)
(d) Translate this passage to Hindi. (Incorrect)
Translate this passage into Hindi. (Correct)
29. (a) I came in a village. (Incorrect)
I came to/into a village. (Correct)
(b) He returns in his town. (Incorrect)
He returns to his town. (Correct)
(c) They are joking to each other. (Incorrect)
They are joking with each other. (Correct)
(d) I congratulate you for your success. (Incorrect)
I congratulate you on your success. (Correct)
30. (a) I have no experience to teach. (Incorrect)
I have no experience of/in teaching. (Correct)
(b) He entered in the house. (Incorrect)
He entered the house. (Correct)
(c) I told to him to come. (Incorrect)
I told him to come. (Correct)
(d) The workers are at strike. (Incorrect)
The workers are on strike. (Correct)
(e) We come at school at 10. (Incorrect)
We come to school at 10. (Correct)
Note: The preposition 'into' with enter is used when one 'enters into an agreement or a
contract'.

31. Cloth is sold with the metre. (Incorrect)


Cloth is sold by the metre. (Correct)
32. She is confident to win the election. (Incorrect)
She is confident of winning the election. (Correct)
33. Trust with God and do the right. (Incorrect)
Trust in God and do the right. (Correct)
34. We should pray God. (Incorrect)
We should pray to God. (Correct)
35. He has invited me for tea. (Incorrect)
He has invited me to tea. (Correct)
36. He succeeded to do it. (Incorrect)
He succeeded in doing it. (Correct)
37. Send there letters on his address. (Incorrect)
Send there letters to his address. (Correct)
38. (a) I agree to you. (Incorrect)
I agree with you. (Correct)
(b) We agree with your proposal. (Incorrect)
We agree to your proposal. (Correct)
(c) Let us agree with a date. (Incorrect)
Let us agree on a date. (Correct)
Note: One 'agrees with a person', 'agrees on a matter for decision', and agrees to a
suggestion or proposal.

39. (a) He is bad in tennis. (Incorrect)


He is bad at tennis. (Correct)
(b) I am good in history. (Incorrect)
I am good at (= skilful at) history. (Correct)
(c) She is clever in cooking. (Incorrect)
She is clever of cooking. (Correct)
Note: 'Bad at', 'good at', 'skilful at', 'clever at', etc. are correct expressions. 40. (a) I am
disappointed about him. (Incorrect)
I am disappointed with him. (Correct)
(b) We had a discussion to the budget. (Incorrect)
We had a discussion on/about the budget. (Correct)
(c) Divide it in three parts. (Incorrect)
Divide it into three parts. (Correct)
(d) Please explain me the reason. (Incorrect)
Please explain to me the reason. (Correct)
Note: Disappointed with somebody with/at/about something; discussion about, on, of; divide
into; explain to somebody.

41. (a) He is ill from flu. (Incorrect)


He is ill with flu. (Correct)
(b) He has no influence about me. (Incorrect)
He has no influence on me. (Correct)
(c) His father insisted in playing. (Incorrect)
His father insisted on playing. (Correct)
(d) I am not interested on your case. (Incorrect)
I am not interested in your case. (Correct)
42. (a) He was kind with me. (Incorrect)
He was kind to me. (Correct)
(b) I am very impressed on your work. (Incorrect)
I am very impressed with/by your work. (Correct)
(c) Her marriage with Rama did not last long. (Incorrect)
Her marriage to Rama did not last long. (Correct)
(d) He prevented me to go there. (Incorrect)
He prevented me from going there. (Correct)
(e) What is the reason of his departure? (Incorrect)
What is the reason for his departure? (Correct)
(f) She was suffering from cold. (Incorrect)
She was suffering with cold. (Correct)
(g) Can he succeed at business? (Incorrect)
Can he succeed in business? (Correct)
43. He sat under the shade of a tree. (Incorrect)
He sat in the shade of a tree. (Correct)
44. He was married with a beautiful lady. (Incorrect)
He was married to a beautiful lady. (Correct)
Note: 'married to one's work' is also a correct expression.

45. You can't deprive me from my right. (Incorrect)


You can't deprive me of my right. (Correct)
46. He is blind from one eye. (Incorrect)
He is blind of one eye. (Correct)
Note: 'Blind to one's shortcomings' is also a correct expression.

47. I am grateful and thank you from the core of my heart. (Incorrect)
I am grateful to and thank you from the core of my heart. (Correct)
48. He finds fault in my work. (Incorrect)
He finds fault with my work. (Correct)
49. He persisted to go there. (Incorrect)
He persisted in going there. (Correct)
50. He was charged of theft. (Incorrect)
He was charged with theft. (Correct)
51. (a) He accused me for cheating him. (Incorrect)
He accused me of cheating him. (Correct)
(b) The prisoner was accused for murder. (Incorrect)
The prisoner was accused of murder. (Correct)
52. (a) My sister is afraid for dogs. (Incorrect)
My sister is afraid of dogs. (Correct)
(b) She is afraid for being knocked down. (Incorrect)
She is afraid of being knocked down. (Correct)
Note: When 'afraid' is followed by an infinitive, no preposition is used; as, she was afraid to
go near the barking dog.

53. (a) My father was angry at my brother. (Incorrect)


My father was angry with my brother. (Correct)
(b) I am angry with what he said. (Incorrect)
I am angry at what he said. (Correct)
Note: 'Angry with a person', but 'at' something he does or says. 'Angry about something' is
also a correct expression; as, What are you angry about?

54. I don't approve your action. (Incorrect)


I don't approve of your action. (Correct)
Note: 'of' is always added to 'approve'.

55. (a) We arrived to school late. (Incorrect)


We arrived at school late. (Correct)
(b) I arrived at here early. (Incorrect)
I arrived here early. (Correct)
Note: 'Arrive' is followed by 'at' for a small place and by 'in' for a big place. If it is followed
by here, somewhere, anywhere or nowhere, no preposition is used after it.

56. (a) Has your father arrived at home yet? (Incorrect)


Has your father arrived home yet? (Correct)
(b) He is going to home. (Incorrect)
He is going home. (Correct)
(c) She arrived her friend's home. (Incorrect)
She arrived at her friend's home. (Correct)
Note: 'Home' is an adverb. No preposition is used before it. When any determiner (my, your,
his, her, their) or genitive (Tom's, my friend's) comes before it then it becomes a noun. A
preposition is placed before it then.

57. A proud man boasts at his achievement. (Incorrect)


A proud man boasts of his achievement. (Correct)
Note: 'Boast about' is also a correct expression.
58. (a) She is very careful for her health. (Incorrect)
She is very careful of her health. (Correct)
(b) He is quite careless for the danger. (Incorrect)
He is quite careless of the danger. (Correct)
Note: 'of' comes after 'careful' and 'careless'. If 'careful' is followed by an infinitive, no
preposition is used then; as,
Be careful not to drive fast.
He was careful to see that the door was locked up.

59. I ordered for a drink. (Incorrect)


I ordered a drink. (Correct)
60. I have passed in the examination. (Incorrect)
I have passed the examination. (Correct)
61. He reached at my house in time. (Incorrect)
He reached my house in time. (Correct)
Note: 'Reach' is never followed by a preposition.

62. How long should I wait you? (Incorrect)


How long should I wait for you? (Correct)
63. (a) The doctor will cure you from influenza. (Incorrect)
The doctor will cure you of influenza. (Correct)
(b) He was cured for his cold. (Incorrect)
He was cured of his cold. (Correct)
(c) What is the best cure of a cold? (Incorrect)
What is the best cure for a cold? (Correct)
Note: 'To cure' is followed by 'of'. When 'cure' is used as a noun, it takes 'for' after it.

64. (a) Many people die from cancer. (Incorrect)


Many people die of cancer. (Correct)
(b) He died of over-eating. (Incorrect)
He died from over-eating. (Correct)
Note: 'Die of a disease' and 'die from something' are correct expressions.

65. My pen is different to that. (Incorrect)


My pen is different from that. (Correct)
66. She was dressed with black. (Incorrect)
She was dressed in black. (Correct)
67. The glass was filled of milk. (Incorrect)
The glass was filled with milk. (Correct)
68. We were glad for a rest after our long journey. (Incorrect)
We were glad of a rest after our long journey. (Correct)
Note: If 'glad' is followed by an infinitive, no preposition is used then; as,
We were glad to be home again.

69. Your suggestions were very helpful for us. (Incorrect)


Your suggestions were very helpful to us. (Correct)
70. She is very interested with my story. (Incorrect)
She is very interested in my story. (Correct)
71. We are proud for our new car. (Incorrect)
We are proud of our new car. (Correct)
72. (a) He can't live with his small wages. (Incorrect)
He can't live on his small wages. (Correct)
(b) Cows live by grass. (Incorrect)
Cows live on grass. (Correct)
Note: 'Live' is followed by 'on'. When 'live' means to earn a living then it is followed by the
preposition 'by'; as,
He lived by cheating others.

73. He told this thing at my face. (Incorrect)


He told this thing to my face. (Correct)
74. I don't believe what he says. (Incorrect)
I don't believe in what he says. (Correct)
75. You should beg pardon from your teacher. (Incorrect)
You should beg pardon of your teacher. (Correct)
76. Distribute these mangoes between forty students. (Incorrect)
Distribute these mangoes among forty students. (Correct)
77. Would you kindly provide me with a desk to sit? (Incorrect)
Would you kindly provide me with a desk to sit at? (Correct)
78. She is fond to talk. (Incorrect)
She is fond of talking. (Correct)
Note: 'Fond of' is always followed by V + ing structure.

79. Both the sisters resemble with each other. (Incorrect)


Both the sisters resemble each other. (Correct)
80. He has disposed his cycle. (Incorrect)
He has disposed of his cycle. (Correct)
81. I objected his coming late. (Incorrect)
I objected to his coming late. (Correct)
82. The beggar was laughed by the naughty boys. (Incorrect)
The beggar was laughed at by the naughty boys. (Correct)
83. (a) He wrote with ink. (Incorrect)
He wrote in ink. (Correct)
(b) She writes with pencil. (Incorrect)
She writes in pencil. (Correct)
Note: But, the note was written with a pencil/pen; she writes with her left hand.

84. (a) This will be very useful for me. (Incorrect)


This will be very useful to me. (Correct)
(b) This tool is quite useful to my purpose. (Incorrect)
This tool is quite useful for my purpose. (Correct)
Note: 'Useful to a person' but 'for a purpose'.

85. (a) We are sorry about that man. (Incorrect)


We are sorry for that man. (Correct)
(b) We are sorry for your misfortune. (Incorrect)
We are sorry about your misfortune. (Correct)
Note: If 'sorry' is followed by an infinitive, no preposition is used then; as,
We are sorry to hear that you have not been well.

86. (a) Flour is made of wheat. (Incorrect)


Flour is made from wheat. (Correct)
(b) Her dress is made from silk. (Incorrect)
Her dress is made of silk. (Correct)
Note: 'Made of', when the original material is not actually changed, but is merely formed in
some object; 'made from', when one substance is changed into another so that a new
substance may be formed.

87. He married to my sister. (Incorrect)


He married my sister. (Correct)
Note: When 'marry' is used in the active voice, no preposition is used. When 'marry' has no
indirect object, 'get married' is often used; as,
Lalit and Mini got married last year.

88. The teacher is pleased for the boy's progress. (Incorrect)


The teacher is pleased with the boy's progress. (Correct)
Note: When 'pleased' is followed by an infinitive, no preposition is used then; as,
We were pleased to hear that he had succeeded.

89. I can't get rid from my cold. (Incorrect)


I can't get rid of my cold. (Correct)
90. We should take care for our health. (Incorrect)
We should take care of our health. (Correct)
Note: If 'take care' is followed by an infinitive or a clause, no preposition is used; as,
Take care to lock everything up safely.
I shall take care that this does not happen again.

91. (a) The pupils were sitting on their desks. (Incorrect)


The pupils were sitting at their desks. (Correct)
(b) There is no chair for me to sit at. (Incorrect)
There is no chair for me to sit on. (Correct)
(c) Let us sit in this seat for a while. (Incorrect)
Let us sit on this seat for a while. (Correct)
(d) She was sitting on a armchair. (Incorrect)
She was sitting in an armchair. (Correct)
(e) Mr. Malhotra was sitting on the car. (Incorrect)
Mr. Malhotra was sitting in the car. (Correct)
Note: A person sits at a desk, on a chair or a seat (but in an armchair) in a car, in a room.
We 'sit at' something that is placed before us, 'on' something that is beneath us and in
something that is around us or to some extent encloses us.

92. He picked up a quarrel with this furious man. (Incorrect)


He picked a quarrel with this furious man. (Correct)
93. We aim to improve the health of our students. (Incorrect)
We aim at improving the health of our students. (Correct)
94. The soldiers set the building to fire. (Incorrect)
The soldiers set the building on fire. (Correct)
95. I prefer death than dishonour. (Incorrect)
I prefer death to dishonour. (Correct)
96. This medicine is a panacea to all diseases. (Incorrect)
This medicine is a panacea for all diseases. (Correct)
97. (a) No miser likes to part from money. (Incorrect)
No miser likes to part with money. (Correct)
(b) I felt grieved when I parted with my friends. (Incorrect)
I felt grieved when I parted from friends. (Correct)
Note: 'Part with' a thing and 'part from' a person are correct expressions.

98. He met me at the eve of his departure for London. (Incorrect)


He met me on the eve of his departure for London. (Correct)
99. He came to see me in the noon. (Incorrect)
He came to see me at noon. (Correct)
100. The headmaster complemented Prem for his scoring a goal. (Incorrect)
The headmaster complemented Prem on his scoring a goal. (Correct)
101. You should not feel jealous against your friends. (Incorrect)
You should not feel jealous of your friends. (Correct)
102. The master took a great liking for the slave boy. (Incorrect)
The master took a great liking to the slave boy. (Correct)
Note: We say'to have a liking for' but 'to take a liking to'.

103. All the students complained against the new time-table. (Incorrect)
All the students complained of the new time-table. (Correct)
Note: 'Complain' is followed by 'of' when the object of the complaint is a thing, not a person.

104. When Akbar came upon the throne, he was elated. (Incorrect)
When Akbar came to the throne, he was elated. (Correct)
105. Yesterday I came across with a juggler. (Incorrect)
Yesterday I came across a juggler. (Correct)
106. Posters announcing his arrival were stuck all over in Lahore. (Incorrect)
Posters announcing his arrival were stuck all over Lahore. (Correct)
107. He took a fancy for this hat. (Incorrect)
He took a fancy to this hat. (Correct)
108. There is little demand of Japanese goods. (Incorrect)
There is little demand for Japanese goods. (Correct)
109. There is no use for crying over spilt milk. (Incorrect)
There is no use of crying over spilt milk. (Correct)
1. As soon as we reach home then we have our evening tea. (Incorrect)
As soon as we reach home, we have our evening tea. (Correct)
2. No sooner did he reach the station, the train started. (Incorrect)
No sooner did he reach the station than the train started. (Correct)
Note: 'As soon as' is never followed by 'then'. 'No sooner' is followed by 'than'.

3. Hardly I had left my house when it began to rain. (Incorrect)


Hardly had I left my house when it began to rain. (Correct)
4. Scarcely I had left my house when it began to rain. (Incorrect)
Scarcely had I left my house before it began to rain. (Correct)
Note: 'Hardly' is followed by 'when' and 'scarcely' by 'before'.

5. The principal as well the teachers was present there. (Incorrect)


The principal as well as the teachers was present there. (Correct)
Note: 'As well as' is the correct expression.

6. He is so intelligent as his brother is. (Incorrect)


He is as intelligent as his brother is. (Correct)
7. Gold is not as useful as iron. (Incorrect)
Gold is not so useful as iron. (Correct)
Note: We use asas in the affirmative sentence and so as in the negative sentence.

8. She loves you more than me. (Incorrect)


She loves you more than I. (Correct)
She loves you more than she does me.
Note: We should use these hidden words after 'than' and see which form of pronoun suits the
best; as,
You like him better than I (do).
You can do it better than I (can).
He is two years older than I (am).
You like him better than (you like).
ButHis wife was several years younger than himit is an incorrect sentence.

9. Unless you don't work hard, you will fail. (Incorrect)


Unless you work hard, you will fail. (Correct)
If you don't work hard, you will fail.
Note: 'Unless' means 'if not'. So, it is never followed by 'no' or 'not'.

10. (a) He went to Patna because he might see Mr. Mitra. (Incorrect)
He went to Patna in order that he might see Mr. Mitra. (Correct)
(b) He takes medicine because he may get better. (Incorrect)
He takes medicine so that he may get well. (Correct)
Note: 'Because' is used for telling the reason but for conveying the purpose, 'in order that' or
'so that' is used.

11. (a) He called me as a fool. (Incorrect)


He called me a fool. (Correct)
(b) I regard him my brother. (Incorrect)
I regard him as my brother. (Correct)
I consider him my brother.
(c) He was appointed as a clerk. (Incorrect)
He was appointed clerk. (Correct)
Note: Regard, describe, represent, portray, depict, mention, define, treattake 'as' after
them. But name, dub, call, appoint, elect, make, choose, think, considerthey don't take 'as'
or 'be' after them.

12. (a) Because he is rude, therefore he is punished. (Incorrect)


Because he is rude he is punished. (Correct)
(b) Because you are poor, therefore I shall help you. (Incorrect)
Because you are poor, I shall help you. (Correct)
Note: 'Because' and 'therefore' are not used in the same sentence.

13. (a) Supposing if she fails, what will she do? (Incorrect)
Suppose she fails, what will she do? (Correct)
(b) Supposing if she comes late, where will she go? (Incorrect)
Supposing she comes late, where will she go? (Correct)
Note: 'Supposing' and 'if' are not used in the same sentence.

14. (a) Until you work hard, you will improve. (Incorrect)
So long as you work hard, you will improve. (Correct)
(b) Wait here as long as five o'clock. (Incorrect)
Wait here until five o'clock. (Correct)
(c) Until it rains, I won't go. (Incorrect)
As long as it rains, I won't go. (Correct)
Note: 'Until' denotes point of time whereas 'as long as/so long as' denotes period of time.

15. (a) He is good if not better than his brother. (Incorrect)


He is as good as, if not better than, his brother. (Correct)
(b) He is more polite, but not so intelligent as Tom. (Incorrect)
He is more polite than, but not so intelligent as Tom.(Correct)
or
He is more polite than Tom, but not so intelligent.(Correct)
Note: If two adjectivesone is in the positive degree and the other in the comparative
degreecome in the same sentence, two conjunctionsfor positive, asas/soas and for
comparative, comparative degree of adjective + than, are used. A single conjunction is not
used for both the adjectives.

16. (a) If you don't labour then you can't pass. (Incorrect)
If you don't labour, you can't pass. (Correct)
(b) As he entered the room then he met me. (Incorrect)
As he entered the room he met me. (Correct)
(c) When he saw the tiger then he was afraid. (Incorrect)
When he saw the tiger, he was afraid. (Correct)
(d) As you are intelligent, so you will pass the exam. (Incorrect)
As you are intelligent, you will pass the exam. (Correct)
Note: As, when, ifthey take neither 'so' nor 'then' with them. When the conjunction is as,
since, seeing that, etc. the subordinate clause usually comes first. There is less emphasis on
the cause and more emphasis on the result (stated in the main clause).

17. Although he worked hard but he failed. (Incorrect)


Although he worked hard yet he failed. (Correct)
Note: 'Although' is followed by 'yet' and not by 'but'.

18. Work hard lest you may not fail. (Incorrect)


Work hard lest you should fail. (Correct)
Note: 'Lest' is followed by 'should'. We don't use 'not' after it.

19. (a) Mohan said that I am learning French. (Incorrect)


Mohan said, "I am learning French".(Correct)
or
Mohan said that he was learning French.(Correct)
(b) He asked me that where my brother was. (Incorrect)
He asked me where his brother was. (Correct)
(c) He enquired that who had stolen the book. (Incorrect)
He enquired who had stolen the book. (Correct)
(d) He asked me that when the bank closed. (Incorrect)
He asked me when the bank closed. (Correct)
Note: When an interrogative sentence is put into indirect speech, 'that' is not used. Thus it is
wrong to sayHe asked me that what the time was. He enquired that why I was late. We
should sayHe asked me what the time was. He enquired why I was late.

20. He had no other recommendation but his robust health. (Incorrect)


He had no other recommendation than his robust health. (Correct)
21. I asked him to lend me his umbrella and he refused to do so. (Incorrect)
I asked him to lend me his umbrella but he refused to do so. (Correct)
22. Now everyone knows that how he suffered for the sake of the poor. (Incorrect)
Now everyone knows how he suffered for the sake of the poor. (Correct)
23. He seldom or ever takes wine. (Incorrect)
He seldom or never takes wine. (Correct)
24. Both the husband as well as the wife were sent to the prison. (Incorrect)
Both the husband and the wife were sent to the prison. (Correct)
25. (a) I have neither written to her or spoken to her. (Incorrect)
I have neither written to her nor spoken to her. (Correct)
(b) He has either a pen nor a pencil. (Incorrect)
He has either a pen or a pencil. (Correct)
(c) He speaks neither English or French. (Incorrect)
He speaks neither English nor French. (Correct)
Note: 'Neither' is followed by 'nor' and 'either' is followed by 'or'.

26. (a) Either he will obey me or take the consequences. (Incorrect)


Either he will obey me or he will take the consequences. (Correct)
(b) He neither know English nor French. (Incorrect)
He know neither English nor French. (Correct)
(c) He not only cheated me but also my friend. (Incorrect)
He cheated not only me but also my friend. (Correct)
(d) Either the step taken was right or wrong. (Incorrect)
The step taken was either right or wrong. (Correct)
(e) He was both strong in power and will. (Incorrect)
He was strong in both power and will. (Correct)
Note: Eitheror, neithernor, bothand, not onlybut alsothey always remain united.

27. (a) She didn't break it nor lose it. (Incorrect)


She did not break it or lose it. (Correct)
(b) I have never spoken to her nor written to her. (Incorrect)
I have never spoken to her or written to her. (Correct)
Note: 'Not' and 'Never' is followed by 'or'.

28. (a) Come here between ten or twelve. (Incorrect)


Come her between ten and twelve. (Correct)
(b) Both Rama as well as Hari were there. (Incorrect)
Both Rama and Hari were there. (Correct)
Note: 'Between' and 'both' are followed by 'and'.
29. There is no one else than you. (Incorrect)
There is no one else but you. (Correct)
Note: 'Else' is followed by 'but', not by 'than'.
1. (a) I saw an US plane yesterday. (Incorrect)
I saw a US plane yesterday. (Correct)
(b) It was an unique sight. (Incorrect)
It was a unique sight. (Correct)
(c) He is an European. (Incorrect)
He is a European. (Correct)
(d) This is an useful book. (Incorrect)
This is a useful book. (Correct)
(e) He is an university professor. (Incorrect)
He is a university professor. (Correct)
Note: US, unique, European, useful and universityeach of these words starts with 'yu'
sound which is consonant sound. So, they take article 'a' before them.

2. (a) My boots were in a good condition. (Incorrect)


My boots were in good condition. (Correct)
(b) My books were in bad state. (Incorrect)
My books were in a bad state. (Correct)
Note: 'In good/bad condition' is a correct expression whereas 'in a good/bad state' is also a
correct expression.

3. (a) He is in a good health. (Incorrect)


He is in good health. (Correct)
(b) I have a homework to do. (Incorrect)
I have homework to do. (Correct)
(c) The accused asked for a mercy. (Incorrect)
The accused asked for mercy. (Correct)
(d) I have received a news from abroad. (Incorrect)
I have received news from abroad. (Correct)
(e) He gets a good pay. (Incorrect)
He gets good pay. (Correct)
(f) We like to have a perfect safety. (Incorrect)
We like to have perfect safety. (Correct)
(g) I want a soap, please. (Incorrect)
I want some soap, please.(Correct)
or
I want a piece/cake of soap, please.(Correct)
(h) I like a comfortable travel. (Incorrect)
I like comfortable travel. (Correct)
(i) It was a bad weather. (Incorrect)
It was bad weather. (Correct)
(j) Teachers' work is a useful work. (Incorrect)
Teachers' work is useful work. (Correct)
(k) We write on a paper. (Incorrect)
We write on paper. (Correct)
Note: Health, homework, mercy, news, pay, safety, soap, travel, weather, work, paperall
are uncountable nouns, and hence do not take 'a' before them. But salary, journey, climate
and paper (= newspaper) are countable nouns; they take 'a' before them.

4. Don't make noise. (Incorrect)


Don't make a noise. (Correct)
5. He has given me good deal of trouble. (Incorrect)
He has given me a good deal of trouble. (Correct)
6. A honest worker is liked by all. (Incorrect)
An honest worker is liked by all. (Correct)
7. He is M.A. (Incorrect)
He is an M.A. (Correct)
8. Hindus and Sikhs should co-operate with one another. (Incorrect)
The Hindus and the Sikhs should co-operate with one another. (Correct)
9. He caught me by neck. (Incorrect)
He caught me by the neck. (Correct)
10. United States of America is a very rich country. (Incorrect)
The United States of America is a rich country. (Correct)
11. The dancing is a popular pastime in Europe. (Incorrect)
Dancing is a popular pastime in Europe. (Correct)
Note: Participle nouns like reading, working, dancing, singing, etc. do not require the definite
article.

12. The cheerfulness is a virtue which one should cultivate. (Incorrect)


Cheerfulness is a virtue which one should cultivate. (Correct)
Note: Abstract nouns like cheerfulness, honesty, generosity, etc. do not require the definite
article.

13. He has been suffering from the pneumonia for the last three days. (Incorrect)
He has been suffering from pneumonia for the last three days. (Correct)
14. My uncle sent me a present on the Christmas day. (Incorrect)
My uncle sent me a present on Christmas day. (Correct)
Note: 'Christmas day' is idiomatic.

15. Go to the hell. (Incorrect)


Go to hell. (Correct)
Note: 'The' is not used, with 'Hell', 'Heaven', 'Paradise' and 'Purgatory'.

16. He was in the town when I wired to him. (Incorrect)


He was in town when I wired to him. (Correct)
Note: 'In town' is idiomatic.

17. The historians generally repeat themselves. (Incorrect)


Historians generally repeat themselves. (Correct)
18. The scientists don't believe in superstitions. (Incorrect)
Scientists don't believe in superstitions. (Correct)
Note: Plural common nouns used in a general sense do not require the definite article.

19. This boy is very good at the Persian. (Incorrect)


This boy is very good at Persian. (Correct)
Note: Languages like French, Persian, English, etc. don't require the definite article.

20. I want five bales of the cotton. (Incorrect)


I want five bales of cotton. (Correct)
21. Have you taken the breakfast? (Incorrect)
Have you taken breakfast? (Correct)
22. Usually they take the dinner together. (Incorrect)
Usually they take dinner together. (Correct)
23. It is better to rule in the hell than to serve in the heaven. (Incorrect)
It is better to rule in hell than to serve in heaven. (Correct)
24. The President Barack Obama broadcast a message of hope to England. (Incorrect)
President Barack Obama broadcast a message of hope to England. (Correct)
25. The Lord Byron was a lover of freedom. (Incorrect)
Lord Byron was a lover of freedom. (Correct)
26. He is opposed to the British Imperialism. (Incorrect)
He is opposed to British Imperialism. (Correct)
27. Give the causes of Mutiny of 1857. (Incorrect)
Give the causes of the Mutiny of 1857. (Correct)
28. Cloth is sold by metre. (Incorrect)
Cloth is sold by the metre. (Correct)
29. Describe the achievements of Ashoka Great. (Incorrect)
Describe the achievements of Ashoka the Great. (Correct)
30. You are in wrong. (Incorrect)
You are in the wrong. (Correct)
31. Ganges flows into Bay of Bengal. (Incorrect)
The Ganges flows into the Bay of Bengal. (Correct)
32. (a) I got hundred books. (Incorrect)
I got a hundred books. (Correct)
(b) I have thousand rupees. (Incorrect)
I have a thousand rupees. (Correct)
(c) He has dozen eggs. (Incorrect)
He has a dozen eggs. (Correct)
Note: 'A hundred', 'a thousand' and 'a dozen' are correct expressions.

33. (a) We pray to the God. (Incorrect)


We pray to God. (Correct)
(b) Durga is goddess. (Incorrect)
Durga is a goddess. (Correct)
Note: 'God' never takes the definite article. But god/goddess takes an article.

34. (a) Books that you gave me are very fine. (Incorrect)
The books that you gave me are very fine. (Correct)
(b) Combs that you sell are black. (Incorrect)
The combs that you sell are black. (Correct)
Note: If plural nouns are used in some particular sense then the definite article is used.

35. (a) I have been invited to dinner which is being given to welcome the minister. (Incorrect)
I have been invited to the dinner which is being given to welcome the minister. (Correct)
(b) He gave us good breakfast. (Incorrect)
He gave us a good breakfast. (Correct)
Note: If there is some special meal, an article is used.

36. (a) I have never seen a such pen. (Incorrect)


I have never seen such a pen. (Correct)
(b) What beautiful house he has! (Incorrect)
What a beautiful house he has! (Correct)
Note: If there is a singular countable noun after 'such' and 'what', 'such a' and 'what a' are
introduced.

37. (a) He is seeking an employment. (Incorrect)


He is seeking employment. (Correct)
(b) He is in search of job. (Incorrect)
He is in search of a job. (Correct)
Note: 'Employment' is an uncountable noun. So, it does not take an article before it. But 'job'
is a countable noun. It takes an article before it.

38. (a) I am B.A. (Incorrect)


I am a B.A. (Correct)
(b) He is S.D.O. (Incorrect)
He is an S.D.O. (Correct)
(c) She came in a hour. (Incorrect)
She came in an hour. (Correct)
Note: 'B.A.' has a consonant sound. So, it takes 'a' before it. But 'S.D.O.' and 'hour' have
vowel sounds, so they take 'an' before them.

39. (a) The sun shines by the day, and not by the night. (Incorrect)
The sun shines by day, not by night. (Correct)
(b) He came here on the foot. (Incorrect)
He came here on foot. (Correct)
(c) She went there at the night. (Incorrect)
She went there at night. (Correct)
Note: 'By day', 'by night', 'on foot' and 'at night' are correct expressions; no article is used
in between them.

40. (a) He talked to me by a telephone. (Incorrect)


He talked to me by telephone. (Correct)
(b) We hear radio in the morning. (Incorrect)
We hear the radio in the morning. (Correct)
(c) Do you ever listen to wireless? (Incorrect)
Do you ever listen to the wireless? (Correct)
Note: 'Telephone' is a countable noun but 'by telephone'
is an idiom. 'Radio' and 'wireless' (systems of broadcasting) take 'the' before them.

41. (a) I work in morning. (Incorrect)


I work in the morning. (Correct)
(b) I rest in afternoon. (Incorrect)
I rest in the afternoon. (Correct)
(c) She plays in evening. (Incorrect)
She plays in the evening. (Correct)
Note: 'In the morning/evening/afternoon' is a correct expression.

42. (a) I must give answer to the question. (Incorrect)


I must give an answer to the question. (Correct)
(b) They got into boat. (Incorrect)
They got into a boat. (Correct)
(c) Everybody needs holiday. (Incorrect)
Everybody needs a holiday. (Correct)
(d) I often ride horse. (Incorrect)
I often ride a horse. (Correct)
(e) I had lesson yesterday. (Incorrect)
I had a lesson yesterday. (Correct)
(f) On such cold morning, I don't go out. (Incorrect)
On such a cold morning, I don't go out. (Correct)
(g) He wants to become politician. (Incorrect)
He wants to become a politician. (Correct)
(h) I take rest for an hour. (Incorrect)
I have a rest for an hour.(Correct)
or
I take a rest for an hour.(Correct)
(i) Calcutta is big city. (Incorrect)
Calcutta is a big city. (Correct)
(j) He travels by the boat/car/train. (Incorrect)
He travels by boat/car/train. (Correct)
Note: Answer, boat, holiday, hours, lesson, morning, politician, rest and city are singular
countable nouns. They take a/an before them. But by boat/plane/car/train is an idiom. It does
not take any article.

43. (a) He took a habit of smoking. (Incorrect)


He took the habit of smoking. (Correct)
(b) It is very important in a life of a person. (Incorrect)
It is very important in the life of a person. (Correct)
(c) I have a cat and a cat is black. (Incorrect)
I have a cat and the cat is black. (Correct)
(d) She hit him on a nose. (Incorrect)
She hit him on the nose. (Correct)
(e) A sea was rough. (Incorrect)
The sea was rough. (Correct)
(f) Sky in blue. (Incorrect)
The sky is blue. (Correct)
Note: If any noun indicates a definite thing, it takes 'the' before it.

44. (a) I play piano. (Incorrect)


I play the piano. (Correct)
(b) She likes harmonium. (Incorrect)
She likes the harmonium. (Correct)
(c) He learns a guitar. (Incorrect)
He learns the guitar. (Correct)
Note: If 'play', 'learn' and 'like' are followed by musical instruments, each of those musical
instruments takes 'the' before it.

45. (a) It was not a best model. (Incorrect)


It was not the best model. (Correct)
(b) This is a smallest TV-set they make. (Incorrect)
This is the smallest TV-set they make. (Correct)
(c) This is biggest gramophone we have. (Incorrect)
This is the biggest gramophone we have. (Correct)
Note: The superlative degrees of adjective (= best, smallest, biggest, etc.) take 'the' before
them.

46. (a) A poor always suffer. (Incorrect)


The poor always suffer. (Correct)
(b) Rich are seldom happy. (Incorrect)
The rich are seldom happy. (Correct)
(c) An unemployed is losing hope. (Incorrect)
The unemployed are losing hope. (Correct)
(d) He is collecting money for blind. (Incorrect)
He is collecting money for the blind. (Correct)
Note: Some of the adjectives which take 'the' before them are as follows: the blind, the deaf,
the sick, the mentally ill, the poor, the rich, the old, the dead, the unemployed, the
uneducated, the handicapped, the foolish, the fortunate, etc. All these words have plural
meanings; as,
The dead = The dead people or all dead people

47. Bombay is London of India. (Incorrect)


Bombay is the London of India. (Correct)
48. Time is best healer. (Incorrect)
Time is the best healer. (Correct)
49. Shyam is better of two. (Incorrect)
Shyam is the better of two. (Correct)
50. The Mt. Everest is the highest peak in the world. (Incorrect)
Mt. Everest is the highest peak in the world. (Correct)
Note: As Mt. Everest is a single mountain, no article is needed.

51. Himalayas are the highest mountains in the world. (Incorrect)


The Himalayas are the highest mountains in the world. (Correct)
Note: As the Himalayas have a range of mountains, the definite article is needed.
52. Rice of Bengal is superior to that of Punjab. (Incorrect)
The rice of Bengal is superior to that of the Punjab. (Correct)
53. Sun was sinking behind hills. (Incorrect)
The sun was sinking behind hills. (Correct)
54. The Valmiki's Ramayana is a well-known religious book. (Incorrect)
Valmiki's Ramayana is a well-known religious book. (Correct)
55. Gita is a sacred book. (Incorrect)
The Gita is a sacred book. (Correct)
Note: The definite article is needed before the names of religious books.

56. The union is strength. (Incorrect)


Union is strength. (Correct)
57. The English is the language of English. (Incorrect)
English is the language of the English. (Correct)
58. The principal Sharma is an experienced man. (Incorrect)
Principal Sharma is an experienced man. (Correct)
59. The first and the last lessons of this book are very interesting. (Incorrect)
The first and last lessons of this book are very interesting.(Correct)
or
The first and the last lesson of this book are very interesting.(Correct)
60. Higher you go, cooler it is. (Incorrect)
The higher you go, the cooler it is. (Correct)
61. Second boy of last row is my friend. (Incorrect)
The second boy of the last row is my friend. (Correct)
62. There are shady trees on the either side of the road. (Incorrect)
There are shady trees on either side of the road. (Correct)
63. The rice grows in Bengal. (Incorrect)
Rice grows in Bengal. (Correct)
64. He has been absent from the college for last two weeks. (Incorrect)
He has been absent from the college for the last two weeks. (Correct)
65. I love to live in country. (Incorrect)
I love to live in the country. (Correct)
66. Singing of the cuckoo is different from singing of the nightingale. (Incorrect)
The singing of the cuckoo is different from the singing of the nightingale. (Correct)
Note: 'Singing' is here particularized, hence the use of the definite article.

67. It is only college where music is taught. (Incorrect)


It is the only college where music is taught. (Correct)
68. Rev. Harper will address the students today. (Incorrect)
The Rev. Harper will address the students today. (Correct)
69. This is more amusing of the two stories. (Incorrect)
This is the more amusing of the two stories. (Correct)
Note: 'The' is used with nouns that represent well-known single objects or single wholes; as,
The sun, the world, the universe, the heavens, the sky, the Bible, the north, the south, the
east, the west, the navy, the bar, the public, the equator.

70. He is now in a trouble. (Incorrect)


He is now in trouble. (Correct)
Note: 'In trouble' is idiomatic.

71. Please allow me to go as I have a work to do at home. (Incorrect)


Please allow me to go as I have work to do at home. (Correct)
Note: 'A work' is un-idiomatic.

72. In spite of an inclement weather, they sailed for their destination. (Incorrect)
In spite of inclement weather, they sailed for their destination. (Correct)
73. Mohan accidentally made a mention of his nephew who is an engineer. (Incorrect)
Mohan accidentally made mention of his nephew who is an engineer. (Correct)
74. He is so unsociable that he has a few friends. (Incorrect)
He is so unsociable that he has few friends. (Correct)
Note: 'A few' means 'some' whereas 'few' means 'none'.
75. He advanced a fallacious reasoning. (Incorrect)
He advanced fallacious reasoning. (Correct)
76. He is a bankrupt. He has a little money. (Incorrect)
He is a bankrupt. He has little money. (Correct)
Note: 'A little' means 'some' whereas 'little' means 'nothing'.
77. There is a vast scope for improvement. (Incorrect)
There is vast scope for improvement. (Correct)
78. (a) All the mankind should be happy. (Incorrect)
All mankind should be happy. (Correct)
(b) We enjoyed the beauty of the Nature. (Incorrect)
We enjoyed the beauty of Nature. (Correct)
(c) We have to learn to live in the society. (Incorrect)
We have to learn to live in society. (Correct)
(d) The man is mortal. (Incorrect)
Man is mortal. (Correct)
Note: 'Mankind', 'nature', 'society' and 'man' are used in general sense. No article is needed.

79. (a) The most people like it. (Incorrect)


Most people like it. (Correct)
(b) The most of the people like it. (Incorrect)
Most of the people like it. (Correct)
Note: If a noun follows 'most' or 'most of', no article is needed before 'most'.

80. Our clock struck the 7 o'clock. (Incorrect)


Our clock struck 7 o'clock. (Correct)
Note: When o'clock is used like a noun, no article is needed.

81. (a) He was given the post of the director. (Incorrect)


He was given the post of director. (Correct)
(b) He accepted the job of the headmaster. (Incorrect)
He accepted the job of headmaster. (Correct)
(c) He had the rank of a captain. (Incorrect)
He had the rank of captain. (Correct)
Note: 'The post/job/rank/position of'nouns following these words do not take any article.
82. (a) I get up before the sunrise. (Incorrect)
I get up before sunrise. (Correct)
(b) He came here after the sunset. (Incorrect) He came here after sunset. (Correct)
Note: Before/after/at sunrise or sunset is a correct expression. No article is required.

83. (a) I want to go to cinema. (Incorrect)


I want to go to the cinema. (Correct)
(b) Do you often go to pictures? (Incorrect)
Do you often go to the pictures? (Correct)
(c) He went to theatre. (Incorrect)
He went to the theatre. (Correct)
(d) She went to movies. (Incorrect)
She went to the movies. (Correct)
(e) He goes to office at ten. (Incorrect)
He goes to the office at ten. (Correct)
Note: Go to the cinema/theatre/pictures/movies/opera/ office/circus is a correct expression.
SayHe is at the office (but at work).

84. (a) I go to the school every day. (Incorrect)


I go to school every day. (Correct)
(b) He goes to the college every day. (Incorrect)
He goes to college every day. (Correct)
Note: When we go to school/college to teach or study, no article is required.

85. The Magistrate passed order asking him to pay a fine. (Incorrect)
The Magistrate passed an order asking him to pay a fine. (Correct)
86. I have little money and I mean to buy few books today. (Incorrect)
I have a little money and I mean to buy a few books today. (Correct)
87. A black and white rabbit were missing. (Incorrect)
A black and a white rabbit were missing. (Correct)
Note: An article is needed before each noun.

88. This soldier receives pension from the Government. (Incorrect)


This soldier receives a pension from the Government. (Correct)
89. His father is very respectable businessman. (Incorrect)
His father is a very respectable businessman. (Correct)
90. He has been avoiding me for the month. (Incorrect)
He has been avoiding me for a month. (Correct)
91. Ramesh is the member of a revolutionary party. (Incorrect)
Ramesh is a member of a revolutionary party. (Correct)
92. The great many people come to receive him. (Incorrect)
A great many people come to receive him. (Correct)
Note: 'A great many' is idiomatic.

93. Use a right word in a right place. (Incorrect)


Use the right word in the right place. (Correct)
94. I know he is a right man for this job. (Incorrect)
I know he is the right man for this job. (Correct)
95. He is a more foolish of the two brothers. (Incorrect)
He is the more foolish of the two brothers. (Correct)
96. The following words are preceded by the definite article; as,
Deccan, Carnatic, Cinema, Punjab, We would saythe Deccan, the Carnatic, the Cinema
and the Punjab.
97. Remember 'the' is used in the following phrases; as,
in the dark, in the wrong, to put to the test
98. No definite article is required in the following verbal and prepositional phrases; as,
to set foot on; to shake hands; to clap hands; to leave town; to leave school; to shorten sail;
to drop anchor; to send word; to lose heart; to take heart; to take oath; to give battle; to do
penance; to lay siege; to follow suit; at sea; in town; at court; at school; at college; in jail; in
bed; out of doors; out of place; on boardship; on deck; in jest; by rule of thumb; by sleight of
hands; to make one's hair stand on end; to call mind; to live from hand to mouth; to have in
hand; to hold/keep at arm's length; short in stature; by word of mouth; love at first sight.
99. (a) Nile is a fine river. (Incorrect)
The Nile is a fine river. (Correct)
(b) Ganga is a holy river. (Incorrect)
The Ganga is a holy river. (Correct)
Note: 'The' is needed before the name of a river.

100. (a) Can you play the tennis? (Incorrect


) Can you play tennis? (Correct)
(b) Do you like the cricket? (Incorrect)
Do you like cricket? (Correct)
(c) Have you ever played a football? (Incorrect)
Have you ever played football? (Correct)
Note: No article is needed before the names of games (tennis/cricket, football, hockey, etc.)

101. (a) He came here at the Christmas. (Incorrect)


He came here at Christmas. (Correct)
(b) The Holi falls in March. (Incorrect)
Holi falls in March. (Correct)
(c) The Deepawali is a happy festival. (Incorrect)
Deepawali is a happy festival. (Correct)
Note: No article is required before the names of festivals.

102. (a) Sports make body strong. (Incorrect)


Sports make the body strong. (Correct)
(b) Games make arms strong. (Incorrect)
Games make the arms strong. (Correct)
(c) Boxing makes back strong. (Incorrect)
Boxing makes the back strong. (Correct)
Note: Parts of the body (body, arms, legs, back, muscles, etc.) take articles before them.

103. (a) He speaks the English. (Incorrect)


He speaks English. (Correct)
(b) Do you know the Hindi? (Incorrect)
Do you know Hindi? (Correct)
Note: No article is needed before the name of a language. But we can saythe English
language or the Hindi language.

104. (a) I saw an one-act play. (Incorrect)


I saw a one-act play. (Correct)
(b) He gave me an one-rupee note. (Incorrect)
He gave me a one-rupee note. (Correct)
(c) I bought an one-way ticket. (Incorrect)
I bought a one-way ticket. (Correct)
Note: 'One' has a consonant sound. That is why it takes 'a', not 'an' before it.

105. (a) He tells lie. (Incorrect)


He tells a lie. (Correct)
(b) He always speaks truth. (Incorrect)
He always speaks the truth. (Correct)
Note: A lie/the truth is a correct expression.
106. (a) He went to U.S.A. (Incorrect)
He went to the U.S.A. (Correct)
(b) She came from U.K. (Incorrect)
She came from the U.K. (Correct)
(c) I returned from U.S.S.R. (Incorrect)
I returned from the U.S.S.R. (Correct)
Note: U.S.A./U.K./U.S.S.R.they take 'the' before them. But no article is required before
America, Britain and Russia. Three Golden Rules

Always bear in mind the following three Golden Rules in make any mistake while using articles:-
(a) Never use 'the' (with plural and uncountable nouns) to talk about things in general; as,
Life is hard. (NotThe life is hard.)
(b) Don't use singular countable nouns without articles; as,
the car or a car (But notcar)
(c) Use a/an to say what people's professions or jobs are; as,
He is a bank manager. (NotHe is bank manager)
In English, each and every word in a sentence has a definite order called word-order. Let us have
a look at the following sentences:-
1. Shakespeare is for me still very difficult. (Incorrect)
Shakespeare is still very difficult for me. (Correct)
2. (a) It was a such cold morning. (Incorrect)
It was such a cold morning. (Correct)
(b) It was so a good pen. (Incorrect)
It was so good a pen. (Correct)
Note: 'Such a' and 'so good a' are correct expressions.

3. (a) Tell me where is the post office. (Incorrect)


Tell me where the post office is. (Correct)
(b) Ask him why does he want a pen. (Incorrect)
Ask him why he wants a pen. (Correct)
Note: In an indirect question, the correct word-order is subject + verb.

4. (a) Do you know where does he live? (Incorrect)


Do you know where he lives? (Correct)
(b) Does he ever think why are they crying? (Incorrect)
Does he ever think why they are crying? (Correct)
Note: Never use a double question (direct question + direct question) in a sentence such a
sentence has a direct question (verb + subject) followed by the indirect question (subject +
verb).

5. (a) I exactly don't know. (Incorrect)


I don't know exactly. (Correct)
(b) He speaks very well English. (Incorrect)
He speaks English very well. (Correct)
(c) We want to go home back. (Incorrect)
We want to go back home. (Correct)
Note: An adverb is placed nearest to the word it qualifies.

6. Please speak to the concerned clerk. (Incorrect)


Please speak to the clerk concerned. (Correct)
Note: 'The concerned clerk', means the worried clerk whereas 'the clerk concerned' means
the 'clerk in charge'.

7. (a) What you are saying? (Incorrect)


What are you saying? (Correct)
(b) Where he is going? (Incorrect)
Where is he going? (Correct)
(c) Why you can't come? (Incorrect)
Why can't you come? (Correct)
(d) Where you went? (Incorrect)
Where did you go? (Correct)
(e) Why she wants to see him? (Incorrect)
Why does she want to see him? (Correct)
Note: In an interrogative sentence, the word-order is which/why/where + helping verb +
subject + main verb.

8. (a) What she wanted to do? (Incorrect)


What did she want to do? (Correct)
(b) Why she did not like it? (Incorrect)
Why did she not like it? (Correct)
(c) Are you know him? (Incorrect)
Do you know him? (Correct)
(d) When do he came? (Incorrect)
When did he come? (Correct)
(e) What does he had? (Incorrect)
What did he has? (Correct)
Note: In an interrogative sentence, the word-orderwhat/ when/why + helping verb +
subject + main verbis set keeping in mind the tense of the sentence.

9. (a) What did she made? (Incorrect)


What did she make? (Correct)
(b) Do you ever seen him? (Incorrect)
Have you ever seen him?(Correct)
or
Do you ever see him?(Correct)
(c) How she was? (Incorrect)
How was she? (Correct)
(d) How is that boat called? (Incorrect)
What is that boat called? (Correct)
(e) If you know him? (Incorrect)
Do you know him? (Correct)
Note: In a direct question, the word-order isverb + subject. Besides, the tense of the verb
(principal + auxiliary) should be correct.
1. Rama has a mind to visit Kulu. (Incorrect)
Rama intends visiting Kulu. (Correct)
2. It was for the first time that Krishna entered head and heart into business. (Incorrect)
It was for the first time that Krishna put heart and soul into business. (Correct)
3. Being failed in the examination he was bitterly disappointed. (Incorrect)
Having failed in the examination, he was bitterly disappointed. (Correct)
4. This is not such a good stick than I expected. (Incorrect)
This is not such a good stick as I expected. (Correct)
5. He prefers this pen than to that pen. (Incorrect)
He prefers this pen to that pen. (Correct)
6. Rama did nothing else to weep. (Incorrect)
Rama did nothing else than weep. (Correct)
7. He has no money with him. (Incorrect)
He has no money on (= about) him. (Correct)
8. My father asked me to go to the fair and enjoy there. (Incorrect)
My father asked me to go to the fair and enjoy myself there. (Correct)
9. You resemble to your mother very much. (Incorrect)
You resemble your mother very much. (Correct)
10. The agitation has been barren off its results. (Incorrect)
The agitation has been barren of results. (Correct)
11. I bow down to your command. (Incorrect)
I bow to your command. (Correct)
12. The one thing which I like about him his honesty. (Incorrect)
The one thing which I like about him is his honesty. (Correct)
13. At the meeting the proposal was thrown off. (Incorrect)
At the meeting the proposal was thrown out. (Correct)
14. Rama Lal is devoted to drinking. (Incorrect)
Rama Lal is addicted to drinking. (Correct)
15. Colonel Rajveer denied to marry Bimla. (Incorrect)
Colonel Rajveer refused to marry Bimla. (Correct)
16. Milton excels all the English poets. (Incorrect)
Milton excels all other English poets. (Correct)
17. The ship was drowned. (Incorrect)
The ship was sunk. (Correct)
18. Do you know swimming? (Incorrect)
Do you know how to swim? (Correct)
19. He wanted to make capital out off this news. (Incorrect)
He wanted to make capital out of this news. (Correct)
20. This is a secret between you and I. (Incorrect)
This is a secret between you and me. (Correct)
21. It is me. (Incorrect)
It is I. (Correct)
22. He was sure it was her. (Incorrect)
He was sure it was she. (Correct)
23. Compare Ashoka to Akbar. (Incorrect)
Compare Ashoka with Akbar. (Correct)
24. A number of houses has collapsed but no loss of life have been reported. (Incorrect)
A number of houses have collapsed but no loss of life has been reported. (Correct)
25. Efforts are being made for engaging a qualified headmaster. (Incorrect)
Efforts are being made to engage a qualified headmaster. (Correct)
26. Since yesterday all the boys refused to attend the classes. (Incorrect)
Since yesterday all the boys have refused to attend the classes. (Correct)
27. I confess that it is I who is to blame. (Incorrect)
I confess that it is I who am to blame. (Correct)
28. If anyone wants to buy this car, they should consult me. (Incorrect)
If anyone wants to buy this car, he should consult me. (Correct)
29. The knives have laid on the table for two hours. (Incorrect)
The knives have lain on the table for two hours. (Correct)
30. I should have liked to have been present at your party. (Incorrect)
I should have liked to be present at your party. (Correct)
31. Plague is raging in the city since the last two months. (Incorrect)
Plague has been raging in the city for the last two months. (Correct)
32. The reason why he is angry is because he is hungry. (Incorrect)
The reason why he is angry is that he is hungry. (Correct)
33. He came late due to an accident. (Incorrect)
He came late owing to an accident. (Correct)
34. The number of the boys is very few today. (Incorrect)
The number of the boys is very small today. (Correct)
35. The murderer was hung at the jail this evening. (Incorrect)
The murderer was hanged at the jail this evening. (Correct)
36. He met with an accident on Thursday, while he drove along Pari Chowk to his residence.
(Incorrect)
He met with an accident on Thursday, while he was driving along Pari Chowk to his residence.
(Correct)
37. This course of action is more preferable than the other one. (Incorrect)
This course of action is more preferable to the other one. (Correct)
38. He was appointed a professor in college, a position which he held for ten years. (Incorrect)
He was appointed to a professorship, a position that he held for ten years. (Correct)
39. Born in Bombay, he went to New York at an early age. (Incorrect)
He was born in Bombay and he went to New York at an early age. (Correct)
40. I could not help but think so. (Incorrect)
I could not help thinking so. (Correct)
41. Bearing in mind the social conditions, it becomes easy to understand the sad plight of the
untouchables. (Incorrect)
When we bear in mind the social conditions, it becomes easy to understand the sad plight of the
untouchables. (Correct)
42. Neither she nor her story were any longer amusing. (Incorrect)
Neither she nor her story was any longer amusing. (Correct)
43. His assistants have and are still doing excellent work. (Incorrect)
His assistants have done and are still doing excellent work. (Correct)
44. This is the third letter I have written and am much surprised at receiving no answer. (Incorrect)
This is the third letter I have written and I am much surprised at receiving no answer. (Correct)
45. Man is unable to fly like a bird nor swim like a fish. (Incorrect)
Man is unable either to fly like a bird or to swim like a fish. (Correct)
46. The proverb teaches to omit no opportunity in doing what is right. (Incorrect)
The proverb teaches us to omit no opportunity of doing what is right. (Correct)
47. Opening the bag, instead of a hare a cat jumped out. (Incorrect)
When he opened the bag, instead of a hare a cat jumped out. (Correct)
48. One of the largest and most representative booksellers are buying five hundred copies of this
book. (Incorrect)
One of the largest and most representative booksellers is buying five hundred copies of this book.
(Correct)
49. The science is opposed to the religion. (Incorrect)
Science is opposed to religion. (Correct)
50. He does not know to behave in the society. (Incorrect)
He does not know how to behave in society. (Correct)
51. The old man had lost the peace of his mind. (Incorrect)
The old man had lost his peace of mind. (Correct)
52. I was totally in dark about the matter. (Incorrect)
I was totally in the dark about the matter. (Correct)
53. It was a case of love at the first sight. (Incorrect)
It was a case of love at first sight. (Correct)
54. The way to the hell is paved with bad intentions. (Incorrect)
The way to hell is paved with bad intentions. (Correct)
55. He is a most popular man in politics. (Incorrect)
He is the most popular man in politics. (Correct)
56. Few months later, he was dismissed. (Incorrect)
A few months later, he was dismissed. (Correct)
57. A very few books are available on the subject. (Incorrect)
Very few books are available on the subject. (Correct)
58. Great many books have been written on this subject. (Incorrect)
A great many books have been written on this subject. (Correct)
59. He is the resident of a village in Multan. (Incorrect)
He is a resident of a village in Multan. (Correct)
60. Last, but not the least, they failed to pay the tax in time. (Incorrect)
Last, but not least, they failed to pay the tax in time. (Correct)
61. You don't understand the human nature. (Incorrect)
You don't understand human nature. (Correct)
62. Pen is mighter than the sword. (Incorrect)
The pen is mighter than the sword. (Correct)
63. These both cases were tried by the same Magistrate. (Incorrect)
Both these cases were tried by the same Magistrate. (Correct)
64. The larger the number of patients, the more the necessity of doctors. (Incorrect)
The larger the number of patients, the greater the necessity of doctors. (Correct)
65. Only a few number of students went to witness the match. (Incorrect)
Only a small number of students went to witness the match. (Correct)
66. Though we were poor, we were honest. (Incorrect)
Though/Although we were poor yet we were honest. (Correct)
67. Nothing is more shocking as the sudden death of a near relation. (Incorrect)
Nothing is more shocking than the sudden death of a near relation. (Correct)
68. To tell in brief, he is innocent in this matter. (Incorrect)
In short, he is innocent in this matter. (Correct)
69. A few months hence I met him in Kolkata. (Incorrect)
A few months ago, I met him in Kolkata.(Correct)
or
A few months later, I met him in Kolkata.(Correct)
70. He is the only best boy in this class. (Incorrect)
He is the best boy in this class. (Correct)
71. The whole India went to see the cricket match. (Incorrect)
The whole of India went to see the cricket match. (Correct)
72. He is more stronger than you. (Incorrect)
He is stronger than you. (Correct)
73. Mohan Lal is a best player of hockey. (Incorrect)
Mohan Lal is a very good player of hockey. (Correct)
74. The officer was too glad to get rid of him. (Incorrect)
The officer was very glad to get rid of him. (Correct)
75. He stuck up to his decision. (Incorrect)
He stuck to his decision. (Correct)
76. You continue to harp on the same worn-out theme. (Incorrect)
You continue to harp on the same worn theme. (Correct)
77. He pointed defects in teaching of English in India. (Incorrect)
He pointed out defects in the teaching of English in India. (Correct)
78. These all apples are rotten. (Incorrect)
All these apples are rotten. (Correct)
79. I am going to Lahore because I may appear in the B.A. examination. (Incorrect)
I am going to Lahore in order that I may appear in the B.A. examination. (Correct)
80. A forbidden fruit is sweat. (Incorrect)
A forbidden fruit is sweet. (Correct)
81. He has a large number of offsprings. (Incorrect)
He has a large number of children. (Correct)
82. Don't find faults with everybody. (Incorrect)
Don't find fault with everybody. (Correct)
83. He had a private business with the Manager. (Incorrect)
He had a piece of private business with the Manager. (Correct)
84. Neither of the professors were consulted. (Incorrect)
Neither of the professors was consulted. (Correct)
85. He is one of those persons who says one thing and does another. (Incorrect)
He is one of those persons who say one thing and do another. (Correct)
86. I have received your letter yesterday. (Incorrect)
I received your letter yesterday. (Correct)
87. His choice of books are praiseworthy. (Incorrect)
His choice of books is praiseworthy. (Correct)
88. Gambling and robbery is the only occupation that interest him. (Incorrect)
Gambling and robbery are the only occupations that interest him. (Correct)
89. It are always hooligans who create disturbance in peaceful meetings. (Incorrect)
It is always hooligans who create disturbance in peaceful meetings. (Correct)
90. Neither I nor Mohan am to blame. (Incorrect)
Neither I nor Mohan is to blame. (Correct)
91. This is one of the theories that has been discredited by scientists. (Incorrect)
This is one of the theories that have been discredited by scientists. (Correct)
92. In this town, nobody want to sell his house. (Incorrect)
In this town, nobody wants to sell his house. (Correct)
93. Ever since that incident, he talked against you. (Incorrect)
Ever since that incident, he has talked against you. (Correct)
94. He has resigned from his post in this office. (Incorrect)
He has resigned his post in this office. (Correct)
95. He has a genius in Algebra. (Incorrect)
He has a genius for Algebra. (Correct)
96. He has no capacity of hard work. (Incorrect)
He has no capacity for hard work. (Correct)
97. He was converted into Christianity. (Incorrect)
He was converted to Christianity. (Correct)
98. He took advantage over my mistake. (Incorrect)
He took advantage of my mistake. (Correct)
99. He is ashamed at his failure in the examination. (Incorrect)
He is ashamed of his failure in the examination. (Correct)
100. I owe an apology to the members of the club of my late coming. (Incorrect)
I owe an apology to the members of the club for my late coming. (Correct)
No part of this publication can be reproduced, stored in a retrieval system or transmitted in any
form or by any means, electronic, mechanical, photocopying, recording or otherwise, without the
prior permission of the author and the publisher.

4/19 Asaf Ali Road,


New Delhi-110 002 (INDIA)
e-mail: prabhatbooks@gmail.com

ISBN 978-93-5048-739-6

Correct English Usage


by Prashant Gupta

Edition
First, 2012

Reserved

You might also like